OBGYN uwise

¡Supera tus tareas y exámenes ahora con Quizwiz!

A 17-year-old female comes to your office for her first gynecologic visit. She has been sexually active for the last year and always uses condoms. What is the most appropriate management regarding Pap smear screening for this patient? A. Pap smear at age 21 B. Pap smear at this visit and then anually C. Pap smear now and then every 3 years E. Pap smear at age 18 D. Pap smear now and then every other year

A. Regardless of cloitarche, Pap smears start at 21

A 72-year-old G3P1 woman has progressive ovarian cancer. She and her husband have already completed a medical power of attorney form. However, the patient did not complete a living will or any other documents expressing her wishes for the initiation of mechanical ventilation or cardioversion in the event of a respiratory or cardiac arrest. Unfortunately, the patient is brought into the hospital after suffering an incapacitating seizure. She is not arousable when she reaches the oncology unit. Her husband Jim is present and willing to act as Mary's surrogate decision-maker. When he decides on the proper course of care, the husband should make decisions based primarily on which of the following? A. What Mary would have chosen B. Mary's best interest C. Hospital Ethics Committee's recommendation D. The family's wishes E. His own wishes

A. A person who has power of attorney should make decisions based on what the patient would have wanted for herself, regardless of what they think her best interests might be.

A 67-year-old G3P3 woman presents with severe pelvic protrusion several years following an abdominal hysterectomy. She denies any incontinence. She failed conservative management with a pessary. As a result, she underwent a vaginal surgical repair where the pubocervical fascia was plicated in the midline, as well as laterally to the arcus tendineus fascia (white line). What defect was repaired in this patient? A. Cystocele B. Rectocele C. Uterine prolapse D. Enterocele E. Urethral diverticulum

A. Central and lateral cystoceles are repaired by fixing defects in the pubocervical fascia or reattaching it to the sidewall, if separated from the white line. Defects in the rectovaginal fascia are repaired in rectoceles. Uterine prolapse is surgically treated by a vaginal hysterectomy, but this patient already had a hysterectomy. Enteroceles are repaired by either vaginal or abdominal enterocele repairs. Vaginal vault prolapse is treated either by supporting the vaginal cuff to the uterosacral or sacrospinous ligaments, or by sacrocolpopexy. Urethral diverticulum does not present with severe pelvic protrusion.

A 27-year-old G1P0 woman at 12 weeks gestation presents for first prenatal care visit. She is previously healthy and takes no medications. An ultrasound is performed and a viable pregnancy is confirmed. At the end of the visit, the patient discusses with you her desire to have a Cesarean section for delivery, as she does not wish to go through the pain of labor. Her husband, an orthopedic surgeon, expresses concerns as they desire to have at least three children and he is worried about potential complications with repeated Cesarean sections. What is the most appropriate next step in the counseling of this patient? A. Agree with her decision after proper counseling and perform a Cesarean section at 39 weeks gestation B. Agree with her decision after proper counseling and perform a Cesarean section at 41 weeks gestation if she has not gone into labor by then C. Advise her that it is not possible to plan a Cesarean section for delivery D. Advise her to listen to her husband and plan for a vaginal birth E. Refer her to psychiatric counseling

A. Elective Cesarean section on demand has been getting more popular among women for a variety of reasons. Although, it might sound unreasonable to undergo a Cesarean section for being afraid of pain, the patient has the right to request it and the physician's duty is to make sure she understands all the risks and potential complications associated with such a decision. Elective delivery should not be scheduled prior to 39 weeks due to risks associated with prematurity. Her husband is appropriately concerned, but it is up to her to make the decision regarding an elective procedure.

A 24-year-old G0 woman is considering pregnancy in the next year. Her medical history and physical examination are normal. She is unaware of any significant family history. She is vegetarian, exercises regularly and does not smoke or drink alcohol. Which of the following is the next best step in the management of this patient? A. Folate supplementation is recommended for this patient B. Most patients get an adequate intake by diet alone C. She is not currently planning a pregnancy and does not need to worry about folate D. Current grain supplementation adequately prevents neural tube defects E. Folate does not need to be started until after a pregnancy is documented

A. Folate lowers homocysteine levels. The Nurses Health Study showed fewer nonfatal MIs and fatal coronary events in women with adequate intake doses of folate and vitamin B6. Folate can also help prevent neural tube defects. Studies have shown that diet alone is not effective in achieving adequate levels, and routine folate supplementation is therefore recommended. Women of reproductive age should take a daily 400-microgram supplement. Adequate levels are especially important prior to pregnancy and during the first four weeks of fetal development. Folic acid levels may be used to diagnose B12 or folate deficiency and are not routinely check to guide folic acid supplementation prior to pregnancy unless a patient is suspected to have a deficiency.

A 48-year-old G3P3 woman presents to the office for a health maintenance examination. Her past medical history is negative. Her family history is significant for hypertension in her father and diabetes mellitus in her mother. Her grandfather passed away from colon cancer at the age of 82. She is worried about getting colon cancer and desires to undergo screening. Her body mass index (BMI) is 23 and her physical exam is normal. What is the most appropriate next step in the management of this patient? A. Recommend a colonoscopy at age 50 and, if normal, repeat every 10 years B. Recommend a colonoscopy at age 50 and then every two years C. Order a colonoscopy now and, if normal, repeat in five years D. Order a sigmoidoscopy now and, if normal, repeat in five years E. Recommend a sigmoidoscopy at age 50 and then every two years

A. For patients with average risk for colon cancer, the recommended screening is to begin colonoscopy at age 50 and then every 10 years, if normal. Despite having a grandfather who passed away from colon cancer, this patient is not necessarily at increased risk and does not need to be screened at different intervals than the general population. If there is a history of a first degree relative with colon cancer before age 60, then begin screening with colonoscopy at age 40, or 10 years before the youngest relative diagnosis, and repeat every five years. Although a sigmoidoscopy can be an acceptable screening procedure, it would still begin at age 50 and repeat every five years, if normal.

A 60-year-old G4P4 woman presents with a two-year history of urine leakage with activity such as coughing, sneezing and lifting. Her past medical history is significant for vaginal deliveries of infants over 9 pounds. She had a previous abdominal hysterectomy and bilateral salpingo-oophorectomy for uterine fibroids. She is on vaginal estrogen for atrophic vaginitis. Physical examination reveals no anterior, apical or posterior wall vaginal prolapse. The vagina is well-estrogenized. Post-void residual is normal. Q-tip test shows a straining angle of 60 degrees from the horizontal. Cough stress test shows leakage of urine synchronous with the cough. Cystometrogram reveals the absence of detrusor instability. The patient failed pelvic muscle exercises and is not interested in an incontinence pessary. Which of the following is the best surgical option for this patient? A. Retropubic urethropexy B. Needle suspension C. Anterior repair D. Urethral bulking procedure E. Colpocleisis

A. Genuine stress incontinence (GSI) is the loss of urine due to increased intra-abdominal pressure in the absence of a detrusor contraction. The majority of GSI is due to urethral hypermobility (straining Q-tip angle >30 degrees from horizon). Some (<10%) of GSI is due to intrinsic sphincteric deficiency (ISD) of the urethra. Patients can have both hypermobility and ISD. Retropubic urethropexy such as tension-free vaginal tape and other sling procedures have the best five-year success rates for patients with GSI due to hypermobility. Needle suspensions and anterior repairs have lower five-year success rates for GSI. Urethral bulking procedures are best for patients with ISD, but with little to no mobility of the urethra. Colpocleisis is one option to treat uterine prolapse, and is not indicated for urinary incontinence.

A 62-year-old G5P5 woman presents with a seven-month history of pelvic pain and pressure, as well as abdominal distention and bloating. She experiences occasional constipation, but no melena or hematochezia. She also has mild to moderate urinary frequency without dysuria, hematuria or flank pain. Her medical history is significant for hypertension and obesity. She went through menopause 12 years ago and has never been on hormone therapy. She reports one episode of light vaginal bleeding several months ago. Her family history is significant for postmenopausal ovarian cancer in her mother and maternal aunt, but is otherwise negative for breast, endometrial or colon cancer. Pelvic examination is remarkable for vaginal atrophy, cervical stenosis and difficult uterine and adnexal assessment due to her body habitus. What is the most appropriate next step in the management of this patient? A. Transvaginal ultrasound B. CT scan of the abdomen and pelvis C. Colonoscopy D. Hysteroscopy E. Diagnostic laparoscopy

A. Given the patient's age, nonspecific abdomino-pelvic symptoms, recent postmenopausal bleeding episode and family history of ovarian cancer, a transvaginal ultrasound is the next best step as it is more sensitive than CT for evaluation of the uterus and adnexa. Colonoscopy is useful for colorectal cancer screening, as well as evaluation of the patient's gastrointestinal symptoms, but would not provide information regarding pelvic anatomy. Diagnostic laparoscopy would be a more invasive procedure that could be performed as indicated, after these other diagnostic studies. Hysteroscopy might be useful based on the ultrasound results, since it might be difficult to perform an endometrial biopsy in the office.

A 22-year-old G0 woman presents with worsening pelvic pain. She previously underwent a laparoscopic ablation of endometriosis followed by continuous oral contraceptive pills. She had short-term relief from this approach, but now has failed this treatment and is seeking additional medical management. Which of the following mechanisms best explains how a gonadotropin releasing hormone (GnRH) agonist would help alleviate her symptoms? A. Down-regulation of the hypothalamic-pituitary gland production B. Up-regulation of the hypothalamic-pituitary gland production C. Suppression of both LH and FSH mid-cycle surges D. Induction of a pseudopregnancy state E. Competitive inhibitor for estrogen receptors

A. Gonadotropin-releasing hormone (GnRH) agonists are analogues of naturally occurring gonadotropin-releasing hormones that down-regulate hypothalamic-pituitary gland production and the release of luteinizing hormone and follicle-stimulating hormone leading to dramatic reductions in estradiol level. Numerous clinical trials show GnRH agonists are more effective than placebo and as effective as Danazol in relieving endometriosis-associated pelvic pain. Danazol, a 17-alpha-ethinyl testosterone derivative, suppresses the mid-cycle surges of LH and FSH. Combined estrogen and progestin therapy in oral contraceptives produces the pseudopregnancy state.

A 36-year-old G0 woman who is a health worker presents for a health maintenance examination. She is sexually active and not using contraception. She doesn't think she is pregnant, but would be happy if she were. As part of her general preventive care, you discuss immunizations. Which vaccination is contraindicated if this patient gets pregnant now? A. Measles-Mumps-Rubella (MMR) B. Pneumococcus C. Hepatitis B D. Polio E. Influenza

A. Pregnancy, or the possibility of pregnancy, within four weeks is a contraindication to the MMR and varicella vaccinations. Tetanus, Hepatitis B, Polio and Pneumococcal vaccinations would not be contraindicated.

A 30-year-old G1P1 woman presents with a history of chronic vulvar pruritus. The itching is so severe that she scratches constantly and is unable to sleep at night. She reports no significant vaginal discharge or dyspareunia. She does not take antibiotics. Her medical history is unremarkable. Pelvic examination reveals normal external genitalia with marked lichenification (increased skin markings) and diffuse vulvar edema and erythema as shown in picture below. Saline microscopy is negative. Potassium hydroxide testing is negative. Vaginal pH is 4.0. The vaginal mucosa is normal. Which of the following is the most likely diagnosis in this patient? A. Lichen simplex chronicus B. Lichen sclerosus C. Lichen planus D. Candidiasis E. Vulvar cancer

A. Lichen simplex chronicus, a common vulvar non-neoplastic disorder, results from chronic scratching and rubbing, which damages the skin and leads to loss of its protective barrier. Over time, a perpetual itch-scratch-itch cycle develops, and the result is susceptibility to infection, ease of irritation and more itching. Symptoms consist of severe vulvar pruritus, which can be worse at night. Clinical findings include thick, lichenified, enlarged and rugose labia, with or without edema. The skin changes can be localized or generalized. Diagnosis is based on clinical history and findings, as well as vulvar biopsy. Treatment involves a short-course of high-potency topical corticosteroids and antihistamines to control pruritus.

A 20-year-old G2P2 healthy woman presents for her post-partum check six weeks after a full term normal spontaneous delivery. She has a 13 month old in addition to the six-week newborn, and is already feeling overwhelmed. She desires a reliable form of contraception. On exam, her vital signs are normal. BMI is 27. The remainder of the exam is unremarkable. Of the following, what is the most effective and appropriate form of contraception for this patient? A. Intrauterine device B. Tubal ligation C. Depo-Provera® D. Oral contraceptive pills E. Essure®

A. Long-acting reversible contraceptives (LARC) methods such as contraceptive implants and intrauterine devices are a good option for this patient. Despite high up-front costs and the need for office visits for insertion and removal, LARC methods provide many distinct advantages over other contraceptive methods as Depo-Provera® and oral contraceptives. While Depo-Provera is an effective form of contraception, it may not be the best choice in this woman with a high BMI. For this young mother who desires a reversible, but reliable form of contraception, the high effectiveness, continuation rate and user satisfaction of LARC methods would be of most benefit. Emerging evidence indicates that increasing the use of LARC methods also could reduce repeat pregnancy among adolescent mothers and repeat abortions among women seeking induced abortion. ("Increasing Use of Contraceptive Implants and Intrauterine Devices To Reduce Unintended Pregnancy," ACOG Committee Opinion, No. 450, 2009). Tubal ligation and Essure® are permanent and are not appropriate for this patient.

A 28-year-old G0 woman presents for preoperative counseling. She has a suspected diagnosis of severe endometriosis and has failed conservative medical management with OCPs. Her symptoms include severe pelvic pain especially prior to and during menstruation, and deep dyspareunia. On pelvic examination, she has uterosacral nodularity and tenderness throughout both adnexa. Ultrasound reveals normal ovaries bilaterally. Which of the following procedure is the best option for this patient? A. Laser ablation B. Ovarian cystectomy C. Salpingectomy D. Bilateral salpingo-oophorectomy E. Total laparoscopic hysterectomy and bilateral salpingo-oophorectomy

A. Surgery is the gold standard in the diagnosis of endometriosis, but often is not the initial treatment as suspected endometriosis is often managed medically. The role of surgery is often to manage the symptoms of endometriosis, often pelvic pain. As such, surgery may be conservative (laparoscopic ablation or excision of implants, excision of endometriomas) or definitive (total hysterectomy/BSO). In this young nulliparous patient, definitive surgery is not indicated, and only laser ablation would be recommended. As she has no evidence of adnexal masses or enlarged ovaries on ultrasound, it is unlikely that ovarian cystectomy would be needed. There is no indication for salpingectomy.

A 28-year-old G0 woman has a pap test which is reported as high-grade squamous intraepithelial lesion (HSIL). She is currently sexually active. She has had six sexual partners and has been in a monogamous relationship with her fiancé for the last year. What is the next most appropriate next step in the management of this patient? A. Colposcopy B. Cryotherapy C. Reflex HPV testing D. Repeat Pap test in one month E. Repeat Pap test in six months

A. The American Society for Colposcopy and Cervical Pathology guidelines recommend immediate LEEP or colposcopy for women with HSIL cytology test results. A diagnostic excisional procedure is recommended for women with HSIL cytology test results when the colposcopic examination is inadequate. Unlike a LEEP, cryotherapy is inadequate as this procedure does not provide a tissue specimen. Repeat cytology testing alone or reflex HPV testing is unacceptable. For women not managed with immediate excision, colposcopy is recommended regardless of HPV result obtained at co-testing.

A 39-year-old G0 woman presents to the clinic reporting non-tender spots on her vulva for about a week. No pruritus or pain is present. She also notes a brownish rash on the palms of her hands. She admits to IV drug abuse. She was diagnosed as HIV-positive two years ago, but has not been compliant with suggested treatment. On examination, three elevated plaques with rolled edges are noted on the vulva. They are non-tender. A brown macular rash is noted on the palms of her hands and the soles of her feet. What is the most appropriate next step in the management of this patient? A. Obtain a treponemal-specific test B. Biopsy of the lesion C. Colposcopic evaluation of the vulvar lesions D. Culture the base of the lesion E. Initiate empiric treatment with doxycycline and ceftriaxone

A. The diagnosis of syphilis is often established by serologic testing. Non-treponemal tests (VDRL or RPR) are non-specific. In this patient with high suspicion for syphilis, specific testing with treponemal antibody can confirm infection. The classic coiled spirochete is easily seen with dark-field microscopy but availability is limited. A characteristic finding is a macular rash on the palms and soles that are often described as copper penny lesions. Colposcopy would not be diagnostic, but certainly is helpful to evaluate for any vulvar lesions thought to be dysplastic. Biopsies can be stained for spirochetes and may show a necrotizing vasculitis, but certainly would not be the most expedient way to make the diagnosis. Penicillin G is the preferred drug for treating all stages of syphilis.

A 26-year-old G2P2 woman presents with a new onset of vulvar burning and irritation. She is sexually active with a new male partner. She is using oral contraception for birth control and did not use a condom. She thought she had a cold about 10 days ago. Which of the following is the most likely diagnosis in this patient? A. Herpes simplex virus B. Primary syphilis C. Secondary syphilis D. Human immunodeficiency virus E. Trichomonas

A. The patient is most likely infected with herpes. Herpes simplex virus is a highly contagious DNA virus. Initial infection is characterized by viral-like symptoms preceding the appearance of vesicular genital lesions. A prodrome of burning or irritation may occur before the lesions appear. With primary infection, dysuria due to vulvar lesions can cause significant urinary retention requiring catheter drainage. Pain can be a very significant finding as well. Treatment is centered on care of the local lesions and the symptoms. Sitz baths, perineal care and topical Xylocaine jellies or creams may be helpful. Anti-viral medications, such as acyclovir, can decrease viral shedding and shorten the course of the outbreak somewhat. These medications can be administered topically or orally. Syphilis is a chronic infection caused by the Treponema pallidum bacterium. Transmission is usually by direct contact with an infectious lesion. Early syphilis includes the primary, secondary, and early latent stages during the first year after infection, while latent syphilis occurs after that and the patient usually has a normal physical exam with positive serology. In primary syphilis, a painless papule usually appears at the site of inoculation. This then ulcerates and forms the chancre, which is a classic sign of the disease. Left untreated, 25% of patients will develop the systemic symptoms of secondary syphilis, which include low-grade fever, malaise, headache, generalized lymphadenopathy, rash, anorexia, weight loss, and myalgias. This patient's symptoms are less consistent with syphilis, but she should still be tested for it. Human immunodeficiency virus is an RNA retrovirus transmitted via sexual contact or sharing intravenous needles. Vulvar burning, irritation or lesions are not typically noted with this disease, although generalized malaise can be. HIV can present with many different signs and symptoms, therefore risk factors should be considered, and testing offered. Trichomonas is a protozoan and is transmitted via sexual contact. It typically presents with a non-specific vaginal discharge. It does not have a systemic manifestation.

A 36-year-old G2P2 woman presents with irregular vaginal bleeding. Six weeks ago, she had her first Depo-Provera® injection and now she has unpredictable bleeding. She is concerned by these symptoms. She has a history of hypertension but is currently on no medications. Vital signs reveal: blood pressure 130/90; weight 188 pounds; height 5 feet 5 inches; BMI 31.4kg/m2. Which of the following is the most appropriate next step in the management of this patient? A. Reassurance B. Begin oral contraceptives C. Begin estrogen D. Insert etonogestrel implant (Implanon) E. Perform an endometrial biopsy

A. The patient should be reassured since initially after Depo-Provera injection there may be unpredictable bleeding. This usually resolves in 2-3 months. In general, after one year of using Depo-Provera, nearly 50% of users have amenorrhea.

You are asked to give a lecture on a new chemotherapy drug that has demonstrated a reasonable efficacy in women with advanced cervical cancer. The day before giving the lecture, you realize that you own stock in the company that makes the drug. Which of the following statements about conflict of interest is true? A. Pharmaceutical companies can support the costs of medical conferences in which physicians receive continuing medical education credits B. b. Physicians should engage in agreements in which companies make a substantial donation to an educational activity, when the donation is contingent on the physician's use or advocacy of a product C. c. The hospital may not interfere with a physician's decision to use a new surgical device D. d. An investigator may not own stock in a company if he/she does research for that company E. e. Physicians are not required to disclose any potential conflict of interest before speaking in a national forum

A. The relationship of physicians and hospitals with pharmaceutical companies is a sensitive one, as there is potential for conflict of interest. It is acceptable for pharmaceutical companies to support conferences in which physicians receive CME credit. Physician participation in those activities should not be contingent upon physician use or advocacy of the product. An investigator may own stock in a company if he/she does research for that company, as long as he/she declares the conflict of interest and the conflict of interest is addressed.

A 48-year-old G0 comes to the office for a health maintenance examination. She is healthy and not taking any medications. She has no history of abnormal Pap tests or sexually transmitted infections. She is not currently sexually active. Her menstrual cycles are normal and her last cycle was three weeks ago. She smokes one pack of cigarettes per day. Her mother was diagnosed with endometriosis and had a hysterectomy and removal of the ovaries at age 38. She is 5 feet 4 inches tall and weighs 130 pounds. On pelvic examination, the patient has a palpable left adnexal mass. An ultrasound was obtained, which showed a 4 cm complex left ovarian cyst and a 2 cm simple cyst on the right ovary. What is the most appropriate next step in the management of this patient? A. Oral contraceptives B. Repeat ultrasound in two months C. CT scan of the abdomen and pelvis D. Needle aspiration of the cyst E. Abdominal hysterectomy and bilateral salpingo-oophorectomy (TAH/BSO)

B. A repeat ultrasound is the most appropriate next step, as this is most likely a hemorrhagic cyst which will resolve on its own. Oral contraceptives are contraindicated in this patient, as she is older than 35 and smokes. A CT scan of the pelvis will not add any more information. Needle aspiration is not the standard of care in this asymptomatic premenopausal patient. There is no indication to proceed with a TAH/BSO.

A 40-year-old G2P2 woman presents for her first health maintenance examination. She denies any new complaints or symptoms. She has no history of any gynecologic problems. Family history is significant for a father with hypertension and a mother, deceased, with breast cancer diagnosed at age 56. A paternal aunt has ovarian cancer which was diagnosed at age 83. A physical exam is unremarkable. What screening test should be offered to this patient next? A. Breast MRI B. Mammogram C. Transvaginal pelvic ultrasound D. Breast ultrasound E. BRCA-1/BRCA-2 testing

B. ACOG recommends that women aged 40 years and older be offered screening mammography annually. Ultrasonography is an established adjunct to mammography. It is useful in evaluating inconclusive mammographic findings, in evaluating young patients and other women with dense breast tissue, and in differentiating a cyst from a solid mass. Breast ultrasound is not recommended as a primary screening modality for women at average risk of developing breast cancer. A combination of first and second-degree relatives on the same side of the family diagnosed with breast and ovarian cancer (one cancer type per person) increases the risk of BRCA mutation. Based on the limited history provided, this patient does not meet the criteria published by ACOG for genetic cancer risk assessment. A more detailed family history regarding risk factors should be obtained to determine whether the patient should be referred for genetic counseling.

A 27-year-old G1P0 woman at 34 weeks gestation is brought in by ambulance after a motor vehicle accident. Although restrained in the car with a safety belt, she suffers a significant head laceration. When she arrives in the emergency department, her initial trauma survey is completed. On her secondary survey, there is bright red blood coming from the vagina. Her abdomen is noted to be tense. Subsequent documentation of the fetal heart tones reveals fetal tachycardia. Abruption is suspected and the patient is rushed to the operating room for an emergent Cesarean section. After delivery, the nurse notes that an informed surgical consent was never signed. Which of the following is true? A. Informed consent is valid if the doctor-patient discussion occurred soon after the patient received intravenous morphine for pain relief B. Informed consent is unnecessary in an emergency situation if a delay in treatment would risk the patient's health/life C. Informed consent is only required for invasive procedures D. Informed consent would not have been valid anyway because the patient sustained a head laceration E. In an emergency situation, informed consent documents can be signed after the procedure is over and the patient is stable

B. Informed consent needs to be obtained for all procedures while patient is fully alert and has not received any narcotics or other medications that may affect her decision-making. The only exception is in true emergency situations that would risk the patient's life. Obtaining informed consent does not necessarily protect the provider from lawsuits and should never be signed after a procedure is already completed.

A 24-year-old G0 woman presents with a one-year history of introital and deep thrust dyspareunia. She has a two-year history of severe dysmenorrhea, despite the use of oral contraceptives. She also reports significant urinary frequency, urgency, and nocturia. A recent urine culture was negative. She underwent a diagnostic laparoscopy six months ago that showed minimal endometriosis with small implants in the posterior cul de sac only, which were ablated with a CO2 laser. What is the most likely diagnosis in this patient? A. Acute cystitis B. Interstitial cystitis C. Acute urethral syndrome D. Acute urethritis E. Salpingitis

B. Interstitial cystitis (IC) is a chronic inflammatory condition of the bladder, which is clinically characterized by recurrent irritative voiding symptoms of urgency and frequency, in the absence of objective evidence of another disease that could cause the symptoms. Pelvic pain is reported by up to 70% of women with IC and, occasionally, it is the presenting symptom or chief complaint. Women may also experience dyspareunia. The specific etiology is unknown, but IC may have an autoimmune and even hereditary component.

A 64-year-old G2P2 woman presents with a 12-month history of severe vulvar pruritus. She has applied multiple over-the-counter topical therapies without improvement. She has no significant vaginal discharge. She has severe dyspareunia at the introitus and has stopped having intercourse because of the pain. Her past medical history is significant for allergic rhinitis and hypertension. On pelvic examination the external genitalia show loss of the labia minora with resorption of the clitoris (phimosis). The vulvar skin appears thin and pale and involves the perianal area as in the picture below. No ulcerations are present. The vagina is mildly atrophic, but appears uninvolved. Which of the following is the most likely diagnosis in this patient? A. Squamous cell hyperplasia B. Lichen sclerosus C. Lichen planus D. Candidiasis E. Vulvar cancer

B. Lichen sclerosus is a chronic inflammatory skin condition that most commonly affects Caucasian premenarchal girls and postmenopausal women. The exact etiology is unknown, but is most likely multifactorial. Patients typically present with extreme vulvar pruritus and may also present with vulvar burning, pain and introital dyspareunia. Early skin changes include polygonal ivory papules involving the vulva and perianal areas, waxy sheen on the labia minora and clitoris, and hypopigmentation. The vagina is not involved. More advanced skin changes may include fissures and erosions due to a chronic itch-scratch-itch cycle, mucosal edema and surface vascular changes. Ultimately, scarring with loss of normal architecture, such as introital stenosis and resorption of the clitoris (phimosis) and labia minora, may occur. Treatment involves use of high-potency topical steroids. There is less than a 5% risk of developing squamous cell cancer within a field of lichen sclerosus.

A 20-year-old G0 college student presents with a one-month history of profuse vaginal discharge and mid-cycle vaginal spotting. She uses oral contraceptives and she thinks her irregular bleeding is due to the pill. She is sexually active and has had a new partner within the past three months. She reports no fevers or lower abdominal pain. She has otherwise been healthy. On pelvic examination, a thick yellow endocervical discharge is noted. Saline microscopy reveals multiple white blood cells, but no clue cells or trichomonads. Potassium hydroxide testing is negative. Vaginal pH is 4.0. No cervical motion tenderness or uterine/adnexal tenderness is present. Testing for gonorrhea and chlamydia is performed, but those results will not be available for several days and the student will be leaving for Europe tomorrow. Which of the following is the most appropriate treatment for this patient? A. Metronidazole and erythromycin B. Ceftriaxone and azithromycin C. Ampicillin and doxycycline D. Azithromycin and doxycycline E. No treatment is necessary until all tests results are known

B. Mucopurulent cervicitis (MPC) is characterized by a mucopurulent exudate visible in the endocervical canal or in an endocervical swab specimen. MPC is typically asymptomatic, but some women have an abnormal discharge or abnormal vaginal bleeding. MPC can be caused by Chlamydia trachomatis or Neisseria gonorrhoeae; however, in most cases neither organism can be isolated. Patients with MPC should be tested for both of these organisms. The results of sensitive tests for C. trachomatis or N. gonorrhoeae (e.g. culture or nucleic acid amplification tests) should determine the need for treatment, unless the likelihood of infection with either organism is high or the patient is unlikely to return for treatment. Antimicrobial therapy should include coverage for both organisms, such as azithromycin or doxycycline for chlamydia and a cephalosporin or quinolone for gonorrhea. Uncomplicated cervicitis, as in this patient, would require only 125 mg of Ceftriaxone in a single dose. Ceftriaxone 250 mg is necessary for the treatment of upper genital tract infection or pelvic inflammatory disease (PID).

A 45-year-old G2P2 woman underwent an abdominal hysterectomy for a large fibroid uterus via a low transverse skin incision. Her postoperative course was significant for new onset right lower quadrant pain and numbness, radiating into the right inguinal area and medial thigh. Her pain was exacerbated by adduction of her right thigh. On abdominal examination, there is a well-healed low transverse incision. Her pain is reproduced with adduction of the right thigh. There is decreased sensation to light touch and pinprick over the right inguinal area and right medial thigh. Patellar reflexes are 2+ and symmetric. Entrapment of which of the following nerves is the most likely cause of her pain? A. Obturator nerve B. Ilioinguinal nerve C. Lateral femoral cutaneous nerve D. Femoral nerve E. Iliohypogastric nerve

B. Nerve entrapment syndrome is a commonly misdiagnosed neuropathy that can complicate pelvic surgical procedures performed through a low transverse incision. The nerves at risk are the iliohypogastric nerve (T-12, L-1) and the ilioinguinal (T-12, L-1) nerve. These two nerves exit the spinal column at the 12th vertebral body and pass laterally through the psoas muscle before piercing the transversus abdominus muscle to the anterior abdominal wall. Once at the anterior superior iliac spine, the iliohypogastric nerve courses medially between the internal and external oblique muscles, becoming cutaneous 1 cm superior to the superficial inguinal ring. The iliohypogastric nerve provides cutaneous sensation to the groin and the skin overlying the pubis. The ilioinguinal nerve follows a similar, although slightly lower, course as the iliohypogastric nerve where it provides cutaneous sensation to the groin, symphysis, labium and upper inner thigh. These nerves may become susceptible to injury when a low transverse incision is extended beyond the lateral border of the rectus abdominus muscle, into the internal oblique muscle. Symptoms are attributed to suture incorporation of the nerve during fascial closure, direct nerve trauma with subsequent neuroma formation, or neural constriction due to normal scarring and healing. Damage to the obturator nerve, which can occur during lymph node dissection would result in the inability of the patient to adduct the thigh.

A 48-year-old G0 woman presents to the office for preoperative counseling. She has severe endometriosis that has failed medical management, and she is planning to undergo a robotic total hysterectomy and salpingo-oophorectomy. She is concerned about developing a "dropped bladder" following her surgery, since both her mother and aunt have undergone surgery for this condition. She reports no urinary incontinence or other urinary or bowel symptoms. She is in good health and exercises with running and weight lifting. Pelvic examination reveals a well-estrogenized vagina, a normal nulliparous cervix, anteverted uterus, and mildly tender adnexa without masses. Which of the following is likely to increase her risk of subsequent development of pelvic organ prolapse? A. Age B. Family History C. Endometriosis D. Exercising E. Hysterectomy

B. Risk factors for the development of pelvic organ prolapse are increasing parity, increasing age, obesity, some connective tissue disorders (Ehlers-Danlos syndrome), and chronic constipation. Vaginal delivery is associated with a higher risk of POP than Cesarean delivery. It is unclear whether occupations that require heavy lifting increase the risk of POP. Women with a family history of POP have up to a 2.5 fold increase in prolapse. Although hysterectomy is associated with an increased risk of apical prolapse, studies show mixed results on the role of hysterectomy in the development of prolapse. The risk of future prolapse may be highest when hysterectomy is performed in women with existing prolapse, while the risk in women with normal pelvic support is less clear.

A 60-year-old G2P2 woman presents with complaints of urinary frequency and urge incontinence. Past medical history is unremarkable. She is on no medications. Pelvic examination reveals no evidence of pelvic relaxation. Post void residual is normal. Urine analysis is negative. A cystometrogram reveals uninhibited detrusor contractions upon filling. Which of the following is the best treatment for this patient? A. Amitriptyline B. Oxybutynin C. Topical (vaginal) estrogen D. Pseudoephedrine E. Kegel exercises

B. The patient has the diagnosis of detrusor instability. The parasympathetic system is involved in bladder emptying and acetylcholine is the transmitter that stimulates the bladder to contract through muscarinic receptors. Thus, anticholinergics are the mainstay of pharmacologic treatment. Oxybutynin is one example. Although the tricyclic antidepressant, amitriptyline, has anticholinergic properties, its side effects do not make it an ideal choice. Vaginal estrogen has been shown to help with urgency, but not urge incontinence. Pseudoephedrine has been shown to have alpha-adrenergic properties and may improve urethral tone in the treatment of stress incontinence. Kegel exercises or pelvic muscle training are used to strengthen the pelvic floor and decrease urethral hypermobility for the treatment of stress urinary incontinence.

A 28-year-old G0 woman presents to the office for a health maintenance examination. She is currently on oral contraceptive pills, but reports a history of irregular menses prior to starting them. Her past medical history is otherwise non-contributory. On physical examination, she is 5 feet 2 inches tall and weighs 180 pounds. She has an area of velvety, hyperpigmented skin on the back of her neck and under her arms. What is the next best step in the management of this patient? A. Complete blood count B. Diabetes screen C. Biopsy pigmented area D. Pelvic ultrasound E. Lipid profile

B. The patient is obese, with a BMI of 33. In addition, the skin changes are consistent with acanthosis nigricans, which is closely associated with insulin resistance. Given these risk factors, she should be tested for diabetes.

A 23-year-old nulliparous woman presents with a painful nodule in her axilla for three days. She is healthy and has no personal or family history of breast disease. On exam, no abnormalities are seen on inspection and no breast mass is palpated. In the axillary area, shaved skin is noted and an erythematous raised 1 cm lesion is palpated and is slightly tender to touch. What is the most likely diagnosis? A. Paget's disease B. Folliculitis C. Fibroadenoma D. Supernumerary nipple E. Duct obstruction

B. This is a typical presentation for folliculitis which can occur with shaving the axillary hair. Paget's disease is a malignant condition of the breast that has the appearance of eczema and does not typically present in the axillary area. Fibroadenomas are common and are usually firm, painless and freely movable. A supernumerary nipple is a congenital variation and is typically located in the nipple line and not tender. A clogged milk duct can be present in the axillary region, but it is typically present in a woman who is breastfeeding.

A 19-year-old G0 woman presents with lower abdominal cramping. The pain started with her menses and has persisted, despite resolution of the bleeding. She thinks she may have a fever, but has not taken her temperature. No urinary frequency or dysuria are present. Her bowel habits are regular. She denies vomiting, but has mild nausea. A yellow blood-tinged vaginal discharge preceded her menses. No pruritus or odor was noted. She is sexually active, uses oral contraceptives and states that her partner does not like condoms. On examination: temperature is 100.2°F (37.9°C); pulse 90; blood pressure 110/60. She is well-developed and nourished and in mild distress. No flank pain is elicited. Her abdomen has normal bowel sounds, but is very tender with guarding in the lower quadrants. No rebound is present. Pelvic examination reveals a moderate amount of thick yellow discharge. The cervix is friable with yellow mucoid discharge at the os. Cervical motion tenderness is present. Uterus and the adnexa are tender without masses. Urine dip is negative for nitrates. Urine pregnancy test is negative. What is the most likely diagnosis? A. Vulvovaginal candidiasis B. Acute salpingitis C. Trichomonas vaginitis D. Cervicitis E. Bacterial vaginosis

B. This patient has findings suggestive of acute salpingitis (pelvic inflammatory disease) including lower abdominal pain, adnexal tenderness, fever, cervical motion tenderness, and vaginal discharge. Mucopurulent cervicitis with exacerbation in the symptoms during and after menstruation is classically gonorrhea. Chlamydia is frequently associated with gonorrhea and also causes cervicitis and pelvic inflammatory disease. Cervicitis alone would not explain this patient's constellation of findings. Trichomonas may cause a yellow frothy discharge, and Candida may cause a thick white cottage cheese-like discharge, but neither would cause fever and abdominal pain.

A 38-year-old G0 woman comes to the office for a health maintenance examination. She is healthy and not taking any medications. She has no history of abnormal Pap tests or sexually transmitted infections. Her menstrual cycles are normal and her last cycle was three weeks ago. Her mother was diagnosed with endometriosis and had a hysterectomy and removal of the ovaries at age 38. She is 5 feet 4 inches tall and weighs 130 pounds. On pelvic examination, the patient has a palpable left adnexal mass. An ultrasound was obtained, which showed a 4 cm complex left ovarian cyst and a 2 cm simple cyst on the right ovary. What is the most likely diagnosis in this patient? A. Endometrioma B. Hemorrhagic cyst C. Ovarian carcinoma D. Mature teratoma E. Polycystic ovaries

B. This patient most likely has a hemorrhagic cyst, considering her history and where she is in her menstrual cycle. Her mother's history of endometriosis does increase her risk; however, it is unlikely since she has never had any symptoms herself. Ovarian carcinoma would need to be ruled out, but it is unlikely in an otherwise asymptomatic premenopausal patient. A mature teratoma would have more pathognomonic findings on ultrasound. This patient does not have typical symptoms, body habitus or ultrasound findings for patients with polycystic ovaries.

A 42-year-old G2P2 woman presents with a two-week history of a thick, curdish white vaginal discharge and pruritus. She has not tried any over-the-counter medications. She is currently single and not sexually active. Her medical history is remarkable for recent antibiotic use for bronchitis. On pelvic examination, the external genitalia show marked erythema with satellite lesions. The vagina appears erythematous and edematous with a thick white discharge. The cervix appears normal and the remainder of the exam is unremarkable except for mild vaginal wall tenderness. Vaginal pH is 4.0. Saline wet prep reveals multiple white blood cells, but no clue cells or trichomonads. Potassium hydroxide prep shows the organisms. Which of the following is the most appropriate treatment for this patient? A. Clindamycin B. Azole cream C. Metronidazole D. Doxycycline E. Ciprofloxacin

B. Vulvovaginal candidiasis (VVC) usually is caused by C. albicans, but is occasionally caused by other Candida species or yeasts. Typical symptoms include pruritus and vaginal discharge. Other symptoms include vaginal soreness, vulvar burning, dyspareunia and external dysuria. None of these symptoms are specific for VVC. The diagnosis is suggested clinically by vulvovaginal pruritus and erythema with or without associated vaginal discharge. The diagnosis can be made in a woman who has signs and symptoms of vaginitis when either: a) a wet preparation (saline or 10% KOH) or Gram stain of vaginal discharge demonstrates yeasts or pseudohyphae; or b) a vaginal culture or other test yields a positive result for a yeast species. Microscopy may be negative in up to fifty percent of confirmed cases. Treatment for uncomplicated VVC consists of short-course topical Azole formulations (1-3 days), which results in relief of symptoms and negative cultures in 80%-90% of patients who complete therapy.

A 29-year-old G0 woman presents due to the inability to conceive for the last 18 months. She has a known history of endometriosis, which was diagnosed by laparoscopy three years ago. She has pelvic pain, which is controlled with non-steroidal anti-inflammatory drugs. Her cycles are regular. She is otherwise in good health and has been married for five years. Her husband had a semen analysis, which was normal. She had a hysterosalpingogram, which showed patent tubes bilaterally. She is getting frustrated that she has not yet achieved pregnancy and asks to proceed with fertility treatments. What is the most appropriate next step in the management of this patient? A. Reassurance and return in six months B. Administer a GnRH agonist C. Ovarian stimulation with clomiphene citrate D. Intrauterine insemination E. Proceed with in vitro fertilization

C. A patient with a known history of endometriosis, who is unable to conceive and has an otherwise negative workup for infertility, benefits from ovarian stimulation with clomiphene citrate, with or without intrauterine insemination. Waiting another six months is not appropriate as she has been trying to conceive for 18 months unsuccessfully. A GnRH agonist is used to control pelvic pain in endometriosis patients unresponsive to other hormonal treatments. In vitro fertilization and adoption can be offered if other treatments fail.

A 76-year-old G3P3 woman presents to your office with worsening urinary incontinence for the past three months. She reports increased urinary frequency, urgency and nocturia. On examination, she has a mild cystocele and rectocele. A urine culture is negative. A post-void residual is 400 cc. Which of the following is the most likely diagnosis in this patient? A. Genuine stress incontinence B. Detrusor instability C. Overflow incontinence D. Functional incontinence E. Mixed incontinence

C. Overflow incontinence is characterized by failure to empty the bladder adequately. This is due to an underactive detrusor muscle (neurologic disorders, diabetes or multiple sclerosis) or obstruction (postoperative or severe prolapse). A normal post-void residual (PVR) is 50-60 cc. An elevated PVR, usually >300 cc, is found in overflow incontinence. Stress incontinence occurs when the bladder pressure is greater than the intraurethral pressure. Overactive detrusor contractions can override the urethral pressure resulting in urine leakage. The mixed variety includes symptoms related to stress incontinence and urge incontinence.

A 32-year-old G2P2 woman presents for a health maintenance examination. She is in good health and has no concerns. She does not have a history of abnormal Pap test and her last one was three years ago. Her examination is normal including her pelvic exam. A Pap test is performed and returns as normal with HPV negative. What is the most appropriate screening recommendation for cervical cancer in this patient? A. Pap test and HPV testing in one year B. Pap test and HPV testing in three years C. Pap test and HPV testing in five years D. HPV testing alone in one year E. HPV testing alone in three years

C. According to the American Cancer Society (ACS), the American Society for Colposcopy and Cervical Pathology (ASCCP), and the American Society for Clinical Pathology (ASCP) guidelines for the Prevention and Early Detection of Cervical Cancer, women ages 30 to 65 years should be screened with cytology and HPV testing (''co-testing'') every five years (preferred) or cytology alone every three years (acceptable). Screening by HPV testing alone is not recommended for most clinical settings and there is insufficient evidence to change screening intervals in this age group following a history of negative screens.

A 32-year-old G0 woman with a last menstrual period three weeks ago, presents with a three-month history of a malodorous vaginal discharge. She reports no pruritus or irritation. She has been sexually active with a new partner for the last four months. Her past medical history is unremarkable. Pelvic examination reveals normal external genitalia without rash, ulcerations or lesions. Some discharge is noted on the perineum. The vagina reveals only a thin, gray homogeneous discharge. The vaginal pH is 5.0. A wet prep is shown in the image below. Which of the following is the most appropriate treatment for this patient? A. Ceftriaxone B. Doxycycline C. Metronidazole D. Azithromycin E. Penicillin

C. Bacterial vaginosis is the most common cause of vaginitis. The infection arises from a shift in the vaginal flora from hydrogen peroxide-producing lactobacilli to non-hydrogen peroxide-producing lactobacilli, which allows proliferation of anaerobic bacteria. The majority of women are asymptomatic; however, patients may experience a thin, gray discharge with a characteristic fishy odor that is often worse following menses and intercourse. Modified Amsel criteria for diagnosis include three out of four of the following: 1) thin, gray homogenous vaginal discharge; 2) positive whiff test (addition of potassium hydroxide releases characteristic amine odor); 3) presence of clue cells on saline microscopy; and 4) elevated vaginal pH >4.5. Treatment consists of Metronidazole 500 mg orally BID for seven days, or vaginal Metronidazole 0.75% gel QHS for five days.

A 90-year-old G7P7 woman presents with severe vaginal prolapse. The entire apex, anterior and posterior wall are prolapsed beyond the introitus. She cannot urinate without reduction of the prolapse. Hydronephrosis is noted on ultrasound of the kidneys and thought to be related to the prolapse. She has a long-standing history of diabetes and cardiac disease. She has failed a trial of pessaries. Which of the following is the next best step in the management of this patient? A. Do nothing and observe B. Anterior and posterior repair C. Colpocleisis D. Sacrospinous fixation E. Sacrocolpopexy

C. Because of the hydronephrosis due to obstruction, intervention is required. Colpocleisis is a procedure where the vagina is surgically obliterated and can be performed quickly without the need for general anesthesia. Anterior and posterior repairs provide no apical support of the vagina. She will be at high risk of recurrent prolapse. The sacrospinous fixation (cuff to sacrospinous-coccygeus complex) or sacrocolpopexy (cuff to sacral promontory using interposed mesh) require regional or general anesthesia and is not the best option for this patient with high surgical morbidity.

A 60-year-old G2P2 postmenopausal woman has an appointment for a health maintenance examination. She wants to discuss bone density screening for osteoporosis. Her medical history is significant for hypertension for the last 10 years that is well controlled with antihypertensive medications. Which of the following is the most appropriate recommendation for this patient regarding initial bone mineral density screening? A. Obtain a DEXA scan now B. Obtain a DEXA scan now only if her blood pressure is not well controlled C. Obtain a DEXA scan at age 65 D. Bone density screening is only necessary if she presents with a fracture E. Recommend bone density screening five years after the onset of menopause

C. Bone density screening is recommended for women beginning at age 65 unless they have pre-existing risk factors which warrant earlier screening. Risk factors for osteoporosis are early menopause, glucocorticoid therapy, sedentary lifestyle, alcohol consumption, hyperthyroidism, hyperparathyroidism, anticonvulsant therapy, vitamin D deficiency, family history of early or severe osteoporosis, or chronic liver or renal disease. These factors would institute early screening in a patient for osteoporosis. A postmenopausal patient presenting with fractures should alert you to suspect osteoporosis.

A 17-year-old G0 female presents with a three-year history of severe dysmenorrhea shortly after menarche at age 14. Her menstrual cycles are regular with heavy flow. She has been treated with ibuprofen and oral contraceptives for the last year without significant improvement. She misses 2-3 days of school each month due to her menses. She has never been sexually active. Physical examination is remarkable for Tanner Stage IV breasts and pubic hair. Pelvic examination is normal, as is a pelvic ultrasound. Both the patient and her mother are concerned. What is the next best step in the management of this patient? A. Sonohysterogram B. CT scan of the pelvis C. Diagnostic laparoscopy D. MRI of the pelvis E. Hysterosalpingogram

C. Chronic pelvic pain is the indication for at least 40% of all gynecologic laparoscopies. Endometriosis and adhesions account for more than 90% of the diagnoses in women with discernible laparoscopic abnormalities, and laparoscopy is indicated in women thought to have either of these conditions. Often, adolescents are excluded from laparoscopic evaluation on the basis of their age, but several series show that endometriosis is as common in adolescents with chronic pelvic pain as in the general population. Therefore, laparoscopic evaluation of chronic pelvic pain in adolescents should not be deferred based on age. Laparoscopy can be both diagnostic and therapeutic in this patient in whom you suspect endometriosis. None of the other imaging modalities listed will help in the further workup of this patient.

A 17-year-old G0 high school student is brought in by her mother for her first gynecologic examination. She began her menses at age 12 and has had regular periods for the past three years. Her last menstrual period was one week ago. For privacy, you ask to examine the patient without her mother. Further history is obtained in the examination room. She admits that she has been sexually active with her boyfriend for the past three years. She uses condoms occasionally and is fearful about possible pregnancy. She requests that her mother not be informed about her sexual activity. On physical examination, she is anxious, but normally developed. Her pelvic examination reveals no vulvar lesions, minimal non-malodorous discharge, and a nulliparous appearing cervix. The bimanual examination reveals a normal size uterus, and her adnexa are non-tender and not enlarged. Urine pregnancy test is negative. In addition to discussing contraception. What is the next best step in the management of this patient? A. Obtain a serum Beta-hCG level B. Obtain a Pap test C. Obtain DNA probes for gonorrhea and chlamydia D. Initiate treatment with doxycycline and ceftriaxone E. Order a pelvic

C. Counseling about and screening for sexually transmitted infections is the best next step. This patient does not require treatment due to a lack of diagnostic criteria. A serum Beta-hCG is not indicated in the setting of normal menstrual cycles with last menstrual period a week ago and a negative urine pregnancy test. Guidelines for initiation of cervical cancer screening is recommended at age 21 regardless of coitarche. A pelvic ultrasound would not be indicated at this time especially since the pregnancy test is negative and given her lack of menstrual or pelvic symptoms.

A 48-year-old G4P4 woman with last menstrual period four weeks ago presents with a one-year history of non-cyclical pelvic pain, dysmenorrhea and dyspareunia. She has a past history of endometriosis, diagnosed 10 years ago by laparoscopy. She had previously been on oral contraceptives for birth control and menstrual cycle regulation, but elected for permanent laparoscopic sterilization 14 months ago. Minimal endometriosis was noted at the time of laparoscopy. She now has recurrent symptoms and desires definitive treatment. Which of the following is the most appropriate surgical option for this patient?A 48-year-old G4P4 woman with last menstrual period four weeks ago presents with a one-year history of non-cyclical pelvic pain, dysmenorrhea and dyspareunia. She has a past history of endometriosis, diagnosed 10 years ago by laparoscopy. She had previously been on oral contraceptives for birth control and menstrual cycle regulation, but elected for permanent laparoscopic sterilization 14 months ago. Minimal endometriosis was noted at the time of laparoscopy. She now has recurrent symptoms and desires definitive treatment. Which of the following is the most appropriate surgical option for this patient? A. Hysteroscopy and dilation and curettage B. Diagnostic laparoscopy C. Hysterectomy with bilateral salpingo-oophorectomy D. Endometrial ablation E. Wedge resection of the ovaries

C. It is estimated that chronic pelvic pain is the principal preoperative indication for 10-12% of hysterectomies. Since the patient had a tubal ligation and does not desire any more children, the best option is removal of ovaries with or without a hysterectomy. Repeat laparoscopy with treatment of endometriosis and adhesions can be helpful; however, the patient will continue to be at increased risk of recurrent disease. An endometrial ablation or wedge resection of ovaries alone would not be very helpful in the setting of non-cyclical pain.

A 26-year-old G0 woman presents to the reproductive endocrinology clinic seeking an infertility evaluation for failing to conceive after 14 months of unprotected intercourse with her boyfriend, who has fathered two other children. She works as a janitor in a nearby elementary school and currently has Medicaid for her health insurance. The physician discourages her from pursuing treatment because she will likely have to pay for her visit with cash, check or charge, and is told that treatment for infertility often involves procedures and technology that are very expensive. She is also informed that, in most states, many of these therapies are not covered by insurance or Medicaid. This situation violates which of the following ethical principles? A. Patient autonomy B. Beneficence C. Justice D. Physician autonomy E. Non-malfeasance

C. Justice requires that we treat like cases alike. It is the physician's duty to educate the patient about all her treatment options in a non-judgmental way regardless of the nature of the treatment and her socioeconomic status.

A 52-year-old G0 woman presents with long-standing vulvar and vaginal pain and burning. She has been unable to tolerate intercourse with her husband because of pain at the introitus. She has difficulty sitting for prolonged periods of time or wearing restrictive clothing because of worsening vulvar pain. She recently noticed that her gums bleed more frequently. She avoids any topical over-the-counter therapies because they intensify her pain. Her physical examination is remarkable for inflamed gingiva and a whitish reticular skin change on her buccal mucosa. A fine papular rash is present around her wrists bilaterally. Pelvic examination reveals white plaques with intervening red erosions on the labia minora as shown in below picture. A speculum cannot be inserted into her vagina because of extensive adhesions. The cervix cannot be visualized. Which of the following is the most likely diagnosis in this patient? A. Squamous cell hyperplasia B. Lichen sclerosus C. Lichen planus D. Genital psoriasis E. Vulvar cancer

C. Lichen planus is a chronic dermatologic disorder involving the hair-bearing skin and scalp, nails, oral mucous membranes and vulva. This disease manifests as inflammatory mucocutaneous eruptions characterized by remissions and flares. The exact etiology is unknown, but is thought to be multifactorial. Vulvar symptoms include irritation, burning, pruritus, contact bleeding, pain and dyspareunia. Clinical findings vary with a lacy, reticulated pattern of the labia and perineum, with or without scarring and erosions as well. With progressive adhesion formation and loss of normal architecture, the vagina can become obliterated. Patients may also experience oral lesions, alopecia and extragenital rashes. Treatment is challenging, since no single agent is universally effective and consists of multiple supportive therapies and topical high potency corticosteroids.

A 36-year-old G2P0 woman at 11 weeks gestational age requests a surgical termination of pregnancy. She had a manual vacuum aspiration last year and would like to undergo the same procedure again. She has chronic hypertension and diabetes well controlled on medications. Vital signs reveal a blood pressure of 120/80 and fasting blood glucose of 100. Which of the following is a contraindication for manual vacuum aspiration of this patient? A. Age B. Parity C. Gestational age D. Chronic hypertension E. Diabetes

C. Manual vacuum aspiration is more than 99% effective in early pregnancy (less than eight weeks). Age, parity and medical illnesses are not contraindications for manual vacuum aspiration. Although the risk of Asherman's syndrome increases with each subsequent pregnancy termination, this patient may still undergo surgical termination as long as she understands risks and benefits. Complications of pregnancy termination increase with increasing gestational age.

A 26-year-old G0 woman returns for a follow-up visit regarding endometriosis. She has been using NSAIDs to manage her pelvic pain, but had to miss four days of work in the last two months. She is sexually active with her husband of two years, although it has been more painful recently. She has regular menstrual cycles and is using condoms for contraception. On pelvic examination, she has localized tenderness in the cul de sac and there are no palpable masses. What is the most appropriate next step in the management of this patient? A. GnRH agonist B. Danazol C. Oral contraceptives D. Laparoscopy and ablation of endometriosis E. Progesterone intrauterine device

C. Oral contraceptives will be the next best choice for this patient. They provide negative feedback to the pituitary-hypothalamic axis which stops stimulation of the ovary resulting in ovarian suppression of sex hormone production, such as estrogen. Since estrogen stimulates endometrial tissue located outside of the endometrium and uterus, endometriosis can be suppressed by OCPs especially when prescribed in a continuous fashion (omit the week of placebo pills resulting in no withdrawal bleed). GnRH agonists also exert negative feedback, but can be used short term only and have more side effects. Danazol is a synthetic androgen used to treat endometriosis, but due to its androgenic side effects (weight gain, increased body hair and acne, and adverse affect on blood lipid levels) it is not usually the first choice of treatment. Laparoscopy is indicated in the patient who fails medical treatment and/or is planning pregnancy in the near future. A progesterone intrauterine device might potentially help alleviate some of her symptoms, but is not the best management for endometriosis.

A 68-year-old G2P2 woman who has recently moved in with her daughter (a long-standing patient of yours) comes in for a health maintenance examination. A vaginal hysterectomy was done in her fifties for uterine prolapse. She is not sure if her ovaries were removed. She has never had an abnormal mammogram or Pap test and has had yearly exams. She stopped hormone replacement therapy 10 years ago. She was recently widowed after being married for 50 years. She does not smoke or drink. Her diabetes is well-controlled with Metformin; she takes a daily baby aspirin and is on a lipid-lowering agent. On examination, she is a thin elderly woman with a dowager's hump. Her breast exam is unremarkable. Her lower genital tract is notable for atrophy. No masses are noted on bimanual and recto-vaginal exam. A fecal occult blood test is negative. Which of the following tests is not necessary? A. Bone density B. Colonoscopy C. Pap test D. Mammogram E. Annual bimanual and recto-vaginal exam

C. Pap test screening is not indicated in patients who have had a hysterectomy, unless it was done for cervical cancer or a high-grade cervical dyspalsia. Patients with a uterus can discontinue cervical cancer screening between the ages of 65-70 if they have had three consecutive negative smears or two negative consecutive cotesting in the last 10 years and no history of high-grade cervical intraepithelial neoplasia or cancer. Patients still need yearly bimanual and rectovaginal exam. Mammograms are done annually, as breast cancer increases with age. Colon cancer screening is recommended at age fifty. The patient has an exaggerated thoracic spine curvature, termed a dowager's hump, likely secondary to thoracic compression fractures secondary to osteoporosis. If this is confirmed on a bone density test, she may benefit from the addition of bisphosphonates

A 25-year-old G3P2 woman, who had recently undergone a primary Cesarean section, had her HIV status revealed to her mother when a nurse left her chart open in the recovery room. She speaks to patient relations and is thinking about seeking damages through legal avenues. When trying to explain the concept of patient privacy, which of the following statements is correct? A. Patient privacy is based on the ethical principle of justice B. Patient privacy is protected by federal law, primarily with the Federal Emergency Medical Treatment and Labor Act (EMTALA) statute C. Patient privacy is the responsibility of physicians; physicians may be fined and/or assessed criminal penalties for violating the privacy of a patient's protected health information D. The patient cannot win a lawsuit in this case because the mother should not have looked at the open record E. Patient privacy is based on the ethical principal of beneficence

C. Patient privacy is the responsibility of physicians. Physicians may be fined and/or assessed criminal penalties for violating the privacy of a patient's protected health information. It was the responsibility of the physicians and the other health care providers in this case to make sure the chart is not left open so someone walking by sees the information.

A 65-year-old G3P3 woman presents with symptoms of vaginal pressure and heaviness, which seem to worsen towards the end of the day. She has a history of three vaginal deliveries. Her surgical history is significant for hysterectomy for abnormal vaginal bleeding at age 45. On exam, she is found to have a large pelvic prolapse. Which of the following is the most appropriate initial treatment of this patient's prolapse? A. Sacrospinous ligament suspension B. Transvaginal tape C. Pessary fitting D. Anterior repair E. Topical vaginal estrogen

C. Pessary fitting is the least invasive intervention for this patient's symptomatic prolapse. Although a sacrospinous ligament suspension would be an appropriate procedure for this patient, it is invasive and not an appropriate first step. Transvaginal tape is used for urinary incontinence and has no role in the management of this patient. An anterior repair can potentially help with her symptoms, depending on what is contributing most to her prolapse but, again, it is invasive. Topical estrogen is unlikely to properly treat her prolapse and related symptoms.

A 23-year-old G0 woman comes to the office to discuss contraception. Her past medical history is remarkable for hypothyroidism and mild hypertension. She has a history of slightly irregular menses. Her best friend recently got a "patch," so she is interested in using a transdermal system (patch). Her vital signs are: blood pressure 130/84; weight 210 pounds; height 5 feet 4 inches. What is the most compelling reason for her to use a different method of contraception? A. Age B. Hypothyroidism C. Weight D. Unpredictable periods E. Her blood pressure

C. The patch has comparable efficiency to the pill in comparative clinical trials, although it has more consistent use. It has a significantly higher failure rate when used in women who weigh more than 198 pounds. The patch is a transdermal system that is placed on a woman's upper arm or torso (except breasts). The patch (Ortho Evra®) slowly releases ethinyl estradiol and norelgestromin, which establishes steady serum levels for seven days. A woman should apply one patch in a different area each week for three weeks, then have a patch-free week, during which time she will have a withdrawal bleed.

A 29-year-old G0 woman presents due to the inability to conceive for the last year. Her cycles are regular every 28 days, but she has very painful periods, occasionally requiring that she miss work despite the use of non-steroidal anti-inflammatory drugs (NSAIDs). She also reports painful intercourse, which is becoming a problem as she now tries to avoid intercourse, even though she would like to conceive. She is otherwise in good health and has been married for five years. She is 5 feet 4 inches tall and weighs 130 pounds. She has a history of pelvic inflammatory disease at age 19, for which she was hospitalized. Her mother had a history of ovarian cancer at age 49. On physical examination, she has abdominal and pelvic lower quadrant tenderness. Uterus is normal in size, but there is a slightly tender palpable left adnexal mass. A pelvic ultrasound shows a 5 cm left complex ovarian cyst and two simple cysts measuring 2 cm in the right ovary. What best explains the underlying pathophysiology of the disease process in this patient? A. Chronic pelvic inflammatory disease B. Family history of ovarian cancer C. Endometrial glands outside the uterine cavity D. Polycystic ovarian syndrome E. Functional hemorrhagic cysts

C. The patient has typical signs of endometriosis which is characterized by the presence of endometrial glands and stroma outside of the uterus. Endometriosis is present in about 30% of infertile woman. She does not have the signs and symptoms of chronic pelvic inflammatory disease. She also does not have the signs and symptoms of polycystic ovarian syndrome, which typically presents with oligomenorrhea in overweight patients. The complex ovarian cyst is most likely an endometrioma (chocolate cyst). The duration of her symptoms makes functional hemorrhagic cyst a less likely option.

A 33-year-old G3P3 woman presents to the office complaining of a new onset vaginal discharge of four days duration. The discharge is thick and white. She has noted painful intercourse and itching since the discharge began. Her vital signs are: blood pressure 120/76 and pulse 78. The pelvic examination reveals excoriations on the perineum, thick white discharge, and is otherwise non-contributory. What is the most likely diagnosis in this patient? A. Herpes simplex virus B. Primary syphilis C. Candida vaginalis D. Bacterial vaginosis E. Trichomonas

C. The patient is most likely has candida vaginalis. Clinically women have itching and thick white cottage cheese like discharge. They may also have burning with urination and pain during intercourse. Herpes simplex viral infections are characterized by viral like symptoms preceding the appearance of vesicular genital lesions. A prodrome of burning or irritation may occur before the lesions appear. With primary infection, dysuria due to vulvar lesions can cause significant urinary retention requiring catheter drainage. Pain can be a very significant finding as well. Syphilis is a chronic infection caused by the Treponema pallidum bacterium. Transmission is usually by direct contact with an infectious lesion. Early syphilis includes the primary, secondary, and early latent stages during the first year after infection, while latent syphilis occurs after that and the patient usually has a normal physical exam with positive serology. In primary syphilis, a painless papule usually appears at the site of inoculation. This then ulcerates and forms the chancre, which is a classic sign of the disease. Left untreated, 25% of patients will develop the systemic symptoms of secondary syphilis, which include low-grade fever, malaise, headache, generalized lymphadenopathy, rash, anorexia, weight loss, and myalgias. Bacterial vaginosis is due to an overgrowth of anaerobic bacteria and characterized by a grayish / opaque foul-smelling discharge. Trichomonas is a protozoan and is transmitted via sexual contact. It typically presents with a non-specific yellow or greenish vaginal discharge. It does not have a systemic manifestation.

A 42-year-old G2P2 woman presents with chronic pelvic pain of two years duration. She describes the pain as constant ever since she underwent a laparoscopic-assisted vaginal hysterectomy for menorrhagia and dysmenorrhea. She did not have any evidence of endometriosis or obvious ovarian pathology at the time of surgery. During the postoperative period, she developed pelvic pain and fever, and was diagnosed with a pelvic/vaginal cuff abscess that was treated with antibiotics and percutaneous drainage. Her pain persisted in the subsequent months. Follow-up imaging over the next two years indicated transient ovarian cysts. Her abdominal examination is notable for mild-moderate tenderness across the lower quadrants, and her pelvic examination is notable for severe tenderness at the vaginal cuff with fullness noted in the midline. Which of the following is the most likely diagnosis in this patient? A. Endometriosis B. Pelvic inflammatory disease C. Pelvic adhesive disease D. Ovarian remnant syndrome E. Ovarian cancer

C. The patient most likely has pelvic adhesive disease as a result of her prior hysterectomy. The development of a postoperative pelvic infection likely has contributed to the further development of pelvic adhesions involving the tubes and ovaries that were retained. Although she did have a pelvic infection, it is unlikely that her pain resulted from classic PID since her adnexa appeared normal at the time of the hysterectomy. It is likely that her tubes and ovaries were affected by the postoperative infection, and as a result she may develop chronic pain from the adhesive disease and tubal damage from that acute infection. The cyclical nature of the ovarian cyst essentially rules out ovarian cancer. Ovarian remnant syndrome occurs following surgical removal of the ovaries, with subsequent development of cyclical pain due to ovarian tissue that was left behind inadvertently.

A 29-year-old G3P0 woman presents for evaluation and treatment of pregnancy loss. Her past medical history is remarkable for three early (< 10 weeks gestation) pregnancy losses and a deep vein thrombosis two years ago. Her work up includes: prolonged dilute Russell viper venom test, elevated anticardiolipin antibodies, normal thyroid function, normal prolactin, and normal MRI of the pelvis. She wishes to get pregnant soon. In addition to aspirin, which of the following treatments is appropriate for this patient? A. No additional treatment B. Corticosteroid C. Heparin D. 17-OH progesterone E. Bromocriptine

C. The prolonged dilute Russell viper venom time, history of three early pregnancy losses, and a history of venous thrombosis leads one to suspect that the etiology of recurrent pregnancy loss is due to antiphospholipid antibody syndrome. The treatment is aspirin plus heparin. There is roughly a 75% success rate with combination therapy versus aspirin alone. There is conflicting evidence regarding steroid use for treatment. 17-OH progesterone is used for the prevention of preterm delivery and not recurrent pregnancy loss. Bromocriptine would be indicated for hyperprolactinemia.

A 74-year-old G0 woman complains of vulvar pain. She reports that the pain is present every day and she has had it for the past year. It now limits her ability to exercise, and she is no longer able to have sexual relations with her partner. On exam, her BMI is 32; blood pressure is 100/60; and heart rate is 77. Her vulva has an ulcerated lesion near the left labial edge. Which of the following is the next best step in the management of this patient? A. Estrogen cream B. Clobetasol cream C. Vulva biopsy D. Laser vaporization of the lesion E. Vulvectomy

C. This patient has a vulvar lesion causing her pain. The next step is to perform a biopsy to evaluate for vulvar cancer. Estrogen cream and clobetasol (a high potency steroid) are treatments for vulvadynia. To diagnosis vulvadynia, all other causes of pain must first be excluded, including infectious etiologies as well as other vulvar conditions. Laser vaporization and vulvectomy are contraindicated until a definitive diagnosis is made.

A 32-year-old G0 woman presents with a one-month history of profuse vaginal discharge with mild odor. She has a new sexual partner with whom she has had unprotected intercourse. She reports mild to moderate irritation, pruritus and pain. She thought she had a yeast infection, but had no improvement after using an over-the-counter antifungal cream. She is concerned about sexually transmitted infections. Her medical history is significant for lupus and chronic steroid use. Pelvic examination shows normal external genitalia, an erythematous vagina with a copious, frothy yellow discharge and multiple petechiae on the cervix. Vaginal pH is 7. Which of the following findings on a wet prep explains the etiology of this condition? A. Hyphae B. Clue cells C. Trichomonads D. Lactobacilli E. Normal epithelial cells

C. This patient has signs and symptoms of trichomoniasis, which is caused by the protozoan, T. vaginalis. Many infected women have symptoms characterized by a diffuse, malodorous, yellow-green discharge with vulvar irritation. However, some women have minimal or no symptoms. Diagnosis of vaginal trichomoniasis is performed by saline microscopy of vaginal secretions, but this method has a sensitivity of only 60% to 70%. The CDC recommended treatment is metronidazole 2 grams orally in a single dose. An alternate regimen is metronidazole 500mg orally twice daily for seven days. The patient's sexual partner also should undergo treatment prior to resuming sexual relations.

A 27-year-old G0 woman presents with a one-year history of dysmenorrhea and dyspareunia. Pain, when present, is 7/10 in strength and requires that she miss work. She now avoids intercourse and no longer finds it pleasurable. She is otherwise in good health. Her last menstrual period was 17 days ago and her menses are typically 28 days apart. She had chlamydia once, at age 19. Physical examination is notable for mild tenderness on abdominal examination in the lower quadrants, and bilateral adnexal tenderness on pelvic examination. Uterus is normal in size and there is uterosacral ligament nodularity. What is the most likely diagnosis in this patient? A. Adenomyosis B. Chronic pelvic inflammatory disease C. Endometriosis D. Endometritis E. Leiomyoma

C. This patient has typical symptoms of endometriosis, including dysmenorrhea and dyspareunia. In addition, the nodularity along the back of the uterus along the uterosacral ligaments is suggestive of endometriosis. Chronic pelvic inflammatory disease would not present this far out from a known infection. Adenomyosis is endometrial glands embedded in the wall of the uterus. Endometritis is an infection of the endometrium. Premenstrual dysphoric disorder (PMDD) is a condition in which a woman has severe depressive symptoms, irritability, and tension before menstruation.

A 57-year-old G2P2 woman presents with a six-month history of urinary incontinence, urgency, and nocturia. She describes the amount of urine loss as large and lasting for several seconds. The urine loss occurs when she is standing or sitting and is not associated with any specific activity. A post-void residual is 50cc. What is the most likely cause of this patient's symptoms? A. Stress incontinence B. Overflow incontinence C. Urge incontinence D. Mixed incontinence E. Vesicovaginal fistula

C. This patient has urge incontinence, which is caused by overactivity of the detrusor muscle resulting in uninhibited contractions, which cause an increase in the bladder pressure over urethral pressure resulting in urine leakage. Stress incontinence is caused by an increase in intra-abdominal pressure (coughing, sneezing) when the patient is in the upright position. This increase in pressure is transmitted to the bladder that then rises above the intra-urethral pressure causing urine loss. Associated structural defects are cystocele or urethrocele. Overflow incontinence is associated with symptoms of pressure, fullness, and frequency, and is usually a small amount of continuous leaking. It is not associated with any positional changes or associated events. Mixed incontinence occurs when increased intra-abdominal pressure causes the urethral-vesical junction to descend causing the detrusor muscle to contract. A vesicovaginal fistula typically results in continuous loss of urine.

A 38-year-old G0 woman comes to the office because she noted a persistent yellow, frothy discharge associated with mild external vulvar irritation. She denies any odor. She tried over the counter anti-fungal medication without success. The discharge has been present for over three months, gradually increasing in amount. Douching has resulted in temporary relief, but the symptoms always recur. Pelvic examination reveals mild erythema at the introitus and a copious yellow frothy discharge fills the vagina. The cervix has erythematous patches on the ectocervix. A sample of the discharge is examined under the microscope. What is the most likely finding? A. Strong amine fishy odor when KOH applied to sample B. Marked polymorphonuclear cells with multi-nucleate giant cells C. Motile ovoid protozoa with flagella D. Budding yeast and pseudo-hyphae E. Clue cells

C. This patient most likely has trichomoniasis. The erythematous patches on the cervix are characteristic of "strawberry cervicitis." Trichomonads are unicellular protozoans, which are easily seen moving across the slide with flagella. The slide must be examined immediately. The discharge is mixed with saline and placed on the slide with a cover slip. Women with trichomonas vaginal infections may have a frothy, yellow-green vaginal discharge. Clue cells are seen on a saline wet mount in women who have bacterial vaginosis. Clue cells are characterized by adherent coccobacillary bacteria that obscure the edges of the cells. A drop of KOH releases amines from the cells and a fishy odor is noted if bacterial vaginosis is present. Yeast vaginitis is characterized by a thick white clumpy discharge which results in erythema, swelling and intense pruritus. Multinucleate giant cells and inflammation may be herpes.

A 37-year-old G0 woman presents with a one-week history of a mildly painful vulvar ulcer. She reports no fevers, malaise or other systemic symptoms. She recently started use of a topical steroid ointment for a vulvar contact dermatitis. She is married and has no prior history of sexually transmitted infections. She reports no travel outside the United States by her husband or herself. Her last Pap smear, six months ago, was normal. A vulvar herpes culture later returns positive for herpes simplex virus type 2. A Rapid Plasma Reagin (RPR) is nonreactive, and HIV testing is negative. Which of the following is the most likely diagnosis in this patient? A. Primary HSV episode B. Recurrent HSV-1 episode C. Recurrent HSV-2 episode D. Atypical HSV episode E. Contact dermatitis

C. Two serotypes of HSV have been identified: HSV-1 and HSV-2. Most cases of recurrent genital herpes are caused by HSV-2. Up to 30% of first-episode cases of genital herpes are caused by HSV-1, but recurrences are much less frequent for genital HSV-1 infection than genital HSV-2 infection. Genital HSV infections are classified as initial primary, initial nonprimary, recurrent and asymptomatic. Initial, or first-episode primary genital herpes is a true primary infection (i.e. no history of previous genital herpetic lesions, and seronegative for HSV antibodies). Systemic symptoms of a primary infection include fever, headache, malaise and myalgias, and usually precede the onset of genital lesions. Vulvar lesions begin as tender grouped vesicles that progress into exquisitely tender, superficial, small ulcerations on an erythematous base. Initial, nonprimary genital herpes is the first recognized episode of genital herpes in individuals who are seropositive for HSV antibodies. Prior HSV-1 infection confers partial immunity to HSV-2 infection and thereby lessens the severity of type 2 infection. The severity and duration of symptoms are intermediate between primary and recurrent disease, with individuals experiencing less pain, fewer lesions, more rapid resolution of clinical lesions and shorter duration of viral shedding. Systemic symptoms are rare. Recurrent episodes involve reactivation of latent genital infection, most commonly with HSV-2, and are marked by episodic prodromal symptoms and outbreaks of lesions at varying intervals and of variable severity. Clinical diagnosis of genital herpes should be confirmed by viral culture, antigen detection or serologic tests. Treatment consists of antiviral therapy with acyclovir, famciclovir or valacyclovir.

A 27-year-old G0 woman presents with a three-year history of dyspareunia. She reports a history of always having painful intercourse, but she is now unable to tolerate intercourse at all. She has avoided sex for the last six months. She describes severe pain with penile insertion. On further questioning, she reports an inability to use tampons because of painful insertion. She also notes a remote history of frequent yeast infections while she was on antibiotics for recurrent sinusitis that occurred years ago. Her medical history is unremarkable, and she is not on medications. Pelvic examination is remarkable for normal appearing external genitalia. Palpation of the vestibule with a Q-tip elicits marked tenderness and slight erythema. A normal-appearing discharge is noted. Saline wet prep shows only a few white blood cells, and potassium hydroxide testing is negative. Vaginal pH is 4.0. The cervix and uterus are unremarkable. Which of the following is the most likely diagnosis in this patient? A. Vaginal cancer B. Genital herpes infection C. Vestibulodynia D. Contact dermatitis E. Chlamydia infection

C. Vestibulodynia (formally vulvar vestibulitis) syndrome consists of a constellation of symptoms and findings limited to the vulvar vestibule, which include severe pain on vestibular touch or attempted vaginal entry, tenderness to pressure and erythema of various degrees. Symptoms often have an abrupt onset and are described as a sharp, burning and rawness sensation. Women may experience pain with tampon insertion, biking or wearing tight pants, and avoid intercourse because of marked introital dyspareunia. Vestibular findings include exquisite tenderness to light touch of variable intensity with or without focal or diffuse erythematous macules. Often, a primary or inciting event cannot be determined. Treatment includes use of tricyclic antidepressants to block sympathetic afferent pain loops, pelvic floor rehabilitation, biofeedback, and topical anesthetics. Surgery with vestibulectomy is reserved for patients who do not respond to standard therapies and are unable to tolerate intercourse.

A 51-year-old G4P4 woman presents for her health maintenance examination. She has not seen a physician for the past two years as she was caring for her sick husband who passed away two months ago. Her last menstrual period was four years ago and she denies any bleeding since that time. Her past medical and surgical histories are negative. Her mother was diagnosed with ovarian cancer and died at age 54. Her Pap smears have always been normal. Her last one was two years ago and it was negative for high-risk HPV types. Her exam is normal. Which of the following is the most appropriate screening test for this patient? A. Pelvic ultrasound B. Endometrial biopsy C. Colonoscopy D. DEXA scan e. Pap smear

C. Women should be offered colorectal cancer screening starting at age 50. Options include yearly hemoccult testing, flexible sigmoidoscopy every five years, or colonoscopy every 10 years. Ultrasound is not a good screening modality for pelvic pathology. An endometrial biopsy is indicated if a patient is experiencing irregular bleeding. A DEXA scan is only recommended in patients with risk factors for osteoporosis prior to age 65. This patient's history does not indicate that she is high risk. A Pap smear is not indicated as she has no recent history of abnormal Pap smears, and her last one with HPV testing was two years ago.

A 36-year-old G3P2 woman presents in active labor at full term with a known placenta previa. She reports brisk vaginal bleeding. Evaluation shows that fetus and patient are currently hemodynamically stable. She has had two normal vaginal deliveries in the past. She declines your recommendation to undergo Cesarean section. Which of the following is not advisable during your initial management of this patient? A. Soliciting her reasons for not undergoing a Cesarean section B. Obtaining hospital Ethics Committee recommendation C. Proceeding with an emergency Cesarean section D. Explaining your reasons for recommending a Cesarean section E. Informing risk management of the situation that has developed

C. You should not perform any procedure on the patient without her consent. It is best in these situations to explain your reasons for the recommended Cesarean section and elicit the patient's reasons for not wanting to undergo the procedure. A court order should only be obtained as a last resort.

Mary is a 65-year-old G2P2 woman with lung metastases from cervical cancer. She was recently weaned from mechanical ventilation after being on the ventilator for four weeks. She has a tracheostomy. Mary currently has worsening pulmonary function and needs to go back on the ventilator or she will die within a few days. Mary's husband, Jim, has power of attorney for Mary's health care decisions. The attending offers Mary a choice of either no ventilation with morphine for comfort or resumption of mechanical ventilation. Mary decides she prefers to go back on the ventilator. Jim prefers that she does not go back on the ventilator because the doctor has said that Mary may never wean off of the ventilator again. Who should make the decision about whether to put Mary back on the ventilator? A. Jim B. All of them together C. Mary's doctor in consultation with Jim D. Mary E. If Jim and Mary cannot agree, consult the hospital ethics committee

D. Since Mary is still competent, she can make her own decisions despite the fact that her husband has power of attorney.

A 68-year-old G3P3 woman comes to the office due to breast tenderness. She is in good health and not taking any medications. Family history is significant for her 70-year-old sister recently diagnosed with breast cancer. On breast examination, her breasts have no lesions; there are no palpable masses, nodules or lymphadenopathy. Her last mammogram was four months ago and was normal. What is the most appropriate next step in the management of this patient? A. Order a mammogram B. Order a breast ultrasound C. Obtain genetic testing (BRCA-1 and BRCA-2 mutations) D. Reassurance E. Order a breast MRI

D. Age and gender are the greatest risk factors for developing breast cancer. Having one first-degree relative with breast cancer does increase the risk. A women's risk of developing breast cancer before menopause is increased if she is BRCA-1 or BRCA-2 positive; however, these genetic mutations occur in a low percentage of the general population. There is no indication for a mammogram since the patient's last mammogram was normal four months ago. Ultrasound and MRI would not add valuable information especially in the setting of a normal mammogram and no masses on physical examination. Genetic testing is not indicated in this case as there is no strong family history and the sister with breast cancer was postmenopausal at time of diagnosis.

A 36-year-old G0 woman presents to the emergency department accompanied by her female partner. The patient notes severe abdominal pain. She states that this pain began 2-3 days ago and was associated with diarrhea as well as some nausea. It has gotten progressively worse and she has now developed a fever. Neither her partner, nor other close contacts, report any type of viral illness. She had her appendix removed as a teenager. On examination, her temperature is 102.0°F (38.9°C), her abdomen is tender with mild guarding and rebound, and she has an elevated white count. On pelvic examination, she is exquisitely tender, such that you cannot complete the examination. Pelvic ultrasound demonstrates bilateral 3-4 cm complex masses. What is the most likely underlying pathogenesis of her illness? A. Diverticulitis B. Gastroenteritis C. Reactivation of an old infection D. Ascending infection E. Pyelonephritis

D. Although salpingitis is most often caused by sexually transmitted agents such as gonorrhea and chlamydia, any ascending infection from the genitourinary tract or gastrointestinal tract can be causative. The infection is polymicrobial consisting of aerobic and anaerobic organisms such as E. coli, Klebsiella, G. vaginalis, Prevotella, Group B streptococcus and/or enterococcus. Although diverticulitis and gastroenteritis should be part of the differential diagnosis initially, the specific findings on examination and ultrasound are more suggestive of bilateral tubo-ovarian abscesses. Even though this patient does not have the typical risk factors for salpingitis, the diagnosis should be considered and explained to the patient in a sensitive and respectful manner. The patient should also be questioned separate from her partner regarding the possibility of other sexual contacts.

A 70-year-old G3P3 woman presents with a four-year history of constant leakage of urine. Her history is significant for abdominal hysterectomy and bilateral salpingo-oophorectomy for endometriosis. She had two anterior repairs in the past for recurrent cystocele. The leakage started six months after her last anterior repair. Pelvic examination reveals no evidence of pelvic relaxation. The vagina is well-estrogenized. Q-tip test reveals a fixed, immobile urethra. Cystometrogram shows no evidence of detrusor instability. Cystourethroscopy showed no evidence of any fistula and reveals a "drain pipe" urethra. Which of the following is the best first treatment for this patient? A. Retropubic urethropexy B. Needle suspension C. Artificial urethral sphincter D. Urethral bulking procedure E. Sling procedure

D. This is a classic example of intrinsic sphincteric deficiency. Urethral bulking procedures are minimally invasive and have a success rate of 80% in these specific patients. The success rates for retropubic urethropexies, needle suspension and slings are less than 50%. An "obstructive or tight" sling can be performed to increase the success rate, but the voiding difficulties are significant, even requiring prolonged or lifelong self-catheterization. Artificial sphincters should be used in patients as a last resort.

A 16-year-old G0 female presents to the emergency department with a two-day history of abdominal pain, nausea and vomiting. She is sexually active with a new partner and is not using any form of contraception. On examination, her temperature is 100.2°F (37.9°C), and she has bilateral lower quadrant pain, with slight rebound and guarding. On pelvic examination, she has purulent cervical discharge and cervical motion tenderness. Her white count is 14,000/mcL. What is the most appropriate next step in the management of this patient? A. Oral amoxicillin clavunate and doxycycline B. Oral metronidazole and doxycycline C. IV metronidazole and doxycycline D. IV cefotetan and doxycycline E. No treatment until culture results are back

D. Although some patients can be treated with an outpatient regimen, this patient should be hospitalized for IV treatment, as she has nausea and vomiting so she might not be able to tolerate oral medications. While adolescents have no better outcomes from inpatient vs outpatient therapy, each patient should be assessed for compliance. It is important to treat aggressively in order to prevent the long-term sequelae of acute salpingitis. You would not wait for culture results before initiating treatment. Her recent sexual contacts should also be informed (by her and/or with her consent) and treated. According to the 2010 CDC treatment guidelines, there are two options for parenteral antibiotics covering both gonorrhea and chlamydia. Cefotetan or cefoxitin PLUS doxycycline or clindamycin PLUS gentamicin. For outpatient treatment, the 2010 CDC guidelines recommend ceftriaxone, cefoxitin, or other third-generation cephalosporin (such as ceftizoxime or cefotaxime) PLUS doxycycline WITH or WITHOUT metronidazole. There are alternative oral regimens as well. http://www.cdc.gov/std/treatment/2010/pid.htm

A 24-year-old G2P2 woman with a history of two prior Cesarean deliveries desires a tubal ligation for permanent sterilization. She has two daughters, who are 3 and 1 years old. She is very sure she does not desire any more children. She is happily married and is a stay-at-home-mom. What is the strongest predictor of post-sterilization regret for this patient? A. Not working outside the home B. Parity C. Marital status D. Age E. Children's gender

D. Approximately 10% of women who have been sterilized regret having had the procedure with the strongest predictor of regret being undergoing the procedure at a young age. The percentage expressing regret was 20% for women less than 30 years old at the time of sterilization. For those under age 25, the rate was as high as 40%. The regret rate was also high for women who were not married at the time of their tubal ligation, when tubal ligation was performed less than a year after delivery, and if there was conflict between the woman and her partner.

A 16-year-old female has a new boyfriend and comes in to discuss contraception. She is well aware of the importance of preventing sexually transmitted infections and specifically wants to know about prevention of pregnancy. Other than abstinence, the most effective method of birth control in this patient is: A. The male condom B. The diaphragm with spermicide C. Oral contraceptives D. Depo-Provera E. The contraceptive ring

D. Contraceptive methods with <1% pregnancy rates (typical use) are Depo-Provera, IUD, sterilization (male or female), and Implanon. Oral contraceptives have a 3-5% pregnancy rate with typical use, and the male condom has a 12% pregnancy rate. Eight percent of women will experience an unintended pregnancy after one year of typical use with a contraceptive ring. Of the methods listed, the diaphragm with spermicide has the highest failure rate (18%) with typical use.

A 23-year-old G0 woman reports having a solitary, painful vulvar lesion that has been present for three days. This lesion has occurred twice in the past. She states that herpes culture was done by her doctor during her last outbreak and was negative. She is getting frustrated in that she does not know her diagnosis. She has no significant previous medical history. She uses oral contraceptives and condoms. She has had four sexual partners in her lifetime. On physical examination, a cluster of three irregular erosions with a superficial crust is noted on the posterior fourchette. Urine pregnancy test is negative. You suspect recurrent genital herpes. How do you explain the negative culture? A. Cultures were taken too early B. Oral contraceptives affect the growth of the virus C. The cultures were refrigerated prior to transport to the lab D. Herpes cultures have a 10-20% false negative rate E. The herpes virus cannot be recovered with recurrent infections

D. Culture is the gold standard in the diagnosis of herpes. They are highly specific, yet sensitivity is limited. It is best to culture the lesion very early in the course. The blister is unroofed and the base is vigorously scraped. The herpes virus can theoretically be isolated from both primary and recurrent infections. This patient very likely presented too late in the course for a useful culture. Oral contraceptives do not affect the growth of viruses. While serum antibody screening can be performed, it indicates lifetime exposure and would not answer the question as to the etiology of the specific lesion. Alternatively, DNA studies such as the polymerase chain reaction can be done, if available.

A 42-year-old G2P2 woman presents for a health maintenance examination. Her past medical history is negative. Her family history is significant for hypertension and hypercholesterolemia in her father and diabetes mellitus in her mother. Her body mass index (BMI) is 23. What lifestyle modification is most important for this patient? A. Starting a weight loss diet B. Starting a sugar-free and cholesterol-free diet C. Recording a daily blood pressure D. Starting an aerobic exercise program E. Recording a weekly blood pressure

D. Heart disease is the number one killer of women. Lifestyle modifications to reduce her risk, especially considering her family history, are important proactive changes that she can make. Studies show an inverse relationship between the level of physical activity and incidence of death from coronary disease. Exercise would be an appropriate first step with this patient. She does not need to lose weight (normal BMI) and does not need to be on a special diet (normal labs). Recording daily or weekly blood pressures is not necessary, but her blood pressure should be checked once a year.

A 37-year-old G3P3 woman presents for contraceptive counseling. She and her husband have decided that they no longer plan to have children and desire permanent sterilization. Her husband refuses to have a vasectomy. On exam, her BMI is 52; blood pressure is 140/80; and heart rate is 86. She has had three previous Cesarean deliveries. Which of the following options would be the be the best method of permanent sterilization? A. Laparoscopic tubal ligation B. Mini-laparotomy with tubal ligation C. Hysterectomy D. Hysteroscopic tubal occlusion (Essure) E. Endometrial ablation

D. Hysteroscopic tubal occlusion is the best option for this patient. Hysteroscopic tubal occlusion (Essure®) can be performed in the office and places coils into the fallopian tubes that cause scarring that blocks the tubes. Patients are required to use a back up method of contraception for three months following the procedure until a hysterosalpingogram is performed confirming complete occlusion of the tubes. While tubal ligation, either by laparoscopy or mini-laparotomy, are common and effective forms of permanent sterilization, for this patient with her BMI and previous surgeries, this would carry more surgical risks. Hysterectomy is not an indicated procedure for sterilization. Endometrial ablation, or thermal destruction of the endometrial tissue, is an effective treatment for menorrhagia but is not reliable for permanent sterilization.

A 32-year-old G3P2 woman with a last menstrual period two weeks ago presents with a six-month history of abdominal pain. She has noncyclic intermittent pain, which she describes as crampy and diffuse across the lower abdomen. Her pain is typically relieved with defecation and is associated with loose, watery stools. Onset of the symptoms is associated with a change in stool frequency from once daily to multiple times daily. She also experiences bloating and abdominal distention several times a week. Her medical history is significant for chronic migraines and she denies previous surgery. Her gynecological history is unremarkable. Her abdominal examination is notable for mild tenderness to palpation in the left lower quadrant, and her pelvic examination is normal. What is the most likely diagnosis in this patient? A. Pelvic adhesions B. Diverticulosis C. Endometriosis D. Irritable bowel syndrome E. C. difficile colitis

D. Irritable bowel syndrome (IBS) is a common functional bowel disorder of uncertain etiology. It is characterized by a chronic, relapsing pattern of abdominal and pelvic pain, and bowel dysfunction with constipation or diarrhea. IBS is one of the most common disorders associated with chronic pelvic pain. IBS appears to occur more commonly in women with chronic pelvic pain than in the general population. Diagnosis is based on the Rome II Criteria for IBS, which includes at least 12 weeks (need not be consecutive) in the preceding 12 months of abdominal discomfort or pain that has two of three features: 1) relief with defecation; 2) onset associated with a change in frequency of stool; or 3) onset associated with a change in stool form or appearance. The patient's history does not support pelvic adhesions, and diverticulosis (although very common) typically may be asymptomatic unless inflammation/infection develops. In this case, the symptoms for IBS may be indistinguishable from diverticulitis or severe diverticular disease. Although severe endometriosis may affect the lower bowel with constricting and invasive implants, the lack of any gynecologic/menstrual symptoms and the normal pelvic examination essentially excludes this diagnosis. The lack of recent antibiotic exposure essentially rules out the diagnosis of C. difficile.

A 32-year-old G3P1 woman presents to your office today because of exposure to hepatitis B. She had vaginal and anal intercourse with a new partner three days ago and did not use condoms. The partner informed her today he was recently diagnosed with acute hepatitis B acquired from intravenous drug use and needle sharing. She has no prior history of hepatitis B infection and has not been vaccinated. She is currently asymptomatic and her examination is normal. Her urine pregnancy test is negative. What is the next best step in the management of this patient? A. Check AST, ALT, and HBsAg B. Administer HBIG one dose C. Administer HBIG two doses D. Administer HBIG and start hepatitis B vaccine series E. Administer hepatitis B vaccine series only

D. It is estimated that 38% of hepatitis B cases worldwide are acquired from sexual transmission. Post-exposure prophlaxis should be inititated as soon as possible but not later than 7 days after blood contact and within 14 days after sexual exposure. In individuals who are unvaccinated but exposed to persons who are HBsAG positive, recommendations are to receive one dose of HBIG (Hepatitis B Immune Globulin) and the HBV (Hepatitis B Vaccine Series). If the source is HBsAG negative or unknown status, then only the HBV series is used. If the exposed individual has been vaccinated and is a responder then no further treatment is necessary. If the exposed individual is vaccinated and a non-responder, then HBIG plus HBV or HBIG times two doses is used. Because the incubation period for the virus is six weeks to six months, checking liver function and immunologic status at this time is not indicated.`

A 25-year-old G1P0 woman at six weeks gestation comes to the office because of undesired pregnancy. You discuss with her the risks and benefits of surgical versus medical abortion using misoprostol and mifepristone. Compared to surgical abortion, which of the following is increased in a woman undergoing a medical abortion? A. Post abortion pain B. Lower failure rate C. Long-term psychological sequelae D. Blood loss E. Future infertility

D. Medical abortion is associated with higher blood loss than surgical abortion. Early in pregnancy (less than 49 days) both medical and surgical procedures can be offered. Mifepristone (an antiprogestin) can be administered, followed by misoprostol (a prostaglandin) to induce uterine contractions to expel the products of conception. This approach has proven to be effective (96%) and safe. A surgical termination is required in the event of failure or excessive blood loss. Medical termination may be more desirable by some patients since they do not have to undergo a surgical procedure. It does not affect future fertility. Any termination of pregnancy, whether medical or surgical, can have psychological sequelae.

A 19-year-old G0 woman presents to the office with a two-week history of low pelvic pain and cramping. She has a new sexual partner and is on oral contraception and uses condoms. She is one week into her cycle. She has noted no vaginal discharge, itch or odor. She denies fevers or chills. She does note that she is on a new diet and has started drinking lots of water. As such, she notes that she is urinating much more frequently. Her examination is entirely unremarkable. Which of the following is the most appropriate next step in the management of this patient? A. Pelvic ultrasound B. Pap test C. Wet prep D. Urinalysis . Testing for chlamydia

D. Mildly symptomatic or asymptomatic urinary tract infections are common in female patients. Urinary tract infection must be considered in patients who present with low pelvic pain, urinary frequency, urinary urgency, hematuria or new issues with incontinence. While yearly screening for chlamydia is recommended for patients less than 25 years old, this patient's symptoms are most consistent with a UTI. A pelvic ultrasound is not indicated at this point.

A 35-year-old G1P0 woman with last menstrual period one week ago presents with an eight-month history of pelvic pain. She reports regular menstrual cycles with moderate flow and dysmenorrhea, relieved with ibuprofen. She describes her pain as a deep, achy sensation with frequent sharp exacerbations. She has not been sexually active for the last several months because of dyspareunia and some arguments with her new partner of one year. She has no history of sexually transmitted infections. Her medical history is significant for irritable bowel syndrome, managed with a fiber supplement. She is a business executive. She has smoked one pack of cigarettes a day since age 25, and drinks a glass of wine three times a week. She tries to exercise regularly by running three to four times a week. This new pain is distinctly different from her IBS symptoms. Which of the following risk factors can contribute to increased incidence of pelvic pain in this patient? A. Alcohol use B. Smoking habit C. Occupation D. New partner E. Age

D. Most published evidence suggests a significant association of physical and sexual abuse with various chronic pain disorders. The arguments with the new partner allude to possible abuse. Studies have found that 40-50% of women with chronic pelvic pain have a history of abuse. Whether abuse (physical or sexual) specifically causes chronic pelvic pain is not clear, nor is a mechanism established by which abuse might lead to the development of chronic pelvic pain. Women with a history of sexual abuse and high somatization scores have been found to be more likely to have non-somatic pelvic pain, suggesting the link between abuse and chronic pelvic pain may be psychologic or neurologic. However, studies also suggest that trauma or abuse may also result in biophysical changes, by literally heightening a person's physical sensitivity to pain. While smoking can be associated with dysmenorrhea, she has been smoking for 10 years and her symptoms are recent onset.

A 23-year-old G1P0 comes into the office after having some light inter-menstrual spotting and cramping. She is currently sexually active and has had unprotected intercourse with two different partners over the past three months. A urine pregnancy test is positive. She does not desire to keep the pregnancy and, after an ultrasound scan in the office reveals a six-week viable intrauterine pregnancy, the patient asks about an abortion, but has no health insurance. What is the most appropriate next step in the management of this patient? A. You inform her that state Medicaid programs are not allowed to cover this service; therefore, you cannot perform the procedure B. You recommend against the procedure due to potential complications with future infertility C. You request she seeks the opinion of both of her partners before undergoing the procedure D. You support her decision for abortion after appropriate counseling E. e. You inform her that abortion should only be performed after six weeks gestation

D. Patients requesting abortion should be counseled appropriately regardless of their insurance status and do not have to obtain the consent of their partner to undergo the procedure. Although there are complications associated with pregnancy termination, they are significantly fewer than complications with carrying a pregnancy. Fewest complications occur when termination is done in the first trimester. From 1990 (the year in which the number of abortions was highest) to 1995, the annual number of legally induced abortions in the United States declined by 15%. Since 1990, factors contributing to the continued decrease in the proportion of pregnancies that ended in abortion might include a decrease in the number of unintended pregnancies, changes in contraceptive practices (including an increased use of condoms among young women), reduced access to abortion services and possible changes in attitudes concerning abortion. Clinical guidelines from the Society of Family Planning state that surgical abortion can be performed successfully and safely as early as three weeks from the onset of last menses if a protocol exists that includes sensitive pregnancy testing, immediate and meticulous examination of the aspirate, and assiduous follow-up of questionable specimens to rule out ectopic pregnancy or continuing gestation.

A 33-year-old G2P2 woman reports a two-year history of severe dysmenorrhea, menorrhagia and pelvic pain following the delivery of her last child. She describes her pelvic pain as primarily in the right lower quadrant, radiating into the vagina. Her pain worsens throughout the day with standing and is associated with pelvic pressure and fullness. Her pelvic examination reveals a mildly enlarged uterus with marked tenderness to palpation of the right adnexa, and no other significant findings. A vaginal ultrasound with color-flow Doppler reveals multiple dilated vessels traversing the right broad ligament to the lower uterus and cervix. The uterus shows no fibroids or other significant changes. Endometrial thickness appears normal. Which of the following is the most likely diagnosis in this patient? A. Endometriosis B. Endometritis C. Adenomyosis D. Pelvic congestion E. Pelvic floor relaxation

D. Pelvic congestion syndrome is a cause of chronic pelvic pain occurring in the setting of pelvic varicosities. The unique characteristics of the pelvic veins make them vulnerable to chronic dilatation with stasis leading to vascular congestion. These veins are thin walled and unsupported, with relatively weak attachments between the supporting connective tissue. The cause of pelvic vein congestion is unknown. Hormonal factors contribute to vasodilatation when pelvic veins are exposed to high concentration of estradiol, which inhibits reflex vasoconstriction of vessels, induces uterine enlargement with selective dilatation of ovarian and uterine veins. This pain may be of variable intensity and duration, is worse premenstrually and during pregnancy, and is aggravated by standing, fatigue and coitus. The pain is often described as a pelvic "fullness" or "heaviness," which may extend to the vulvar area and legs. Associated symptoms include vaginal discharge, backache and urinary frequency. Menstrual cycle defects and dysmenorrhea are common. No signs of pelvic floor relaxation were noted on exam.

A 23-year-old G1 woman with six weeks amenorrhea presents with lower abdominal pain and vaginal bleeding. Her temperature is 102.0°F (38.9°C) and the cervix is 1 cm dilated. Uterus is eight-week size, tender and there are no adnexal masses. Urine pregnancy test is positive. Which of the following is the most appropriate next step in the management of this patient? A. Observation B. Single-agent antibiotics C. Medical termination of pregnancy plus antibiotics D. Uterine evacuation plus antibiotics E. Laparoscopy plus antibiotics

D. The management of septic abortion includes broad-spectrum antibiotics and uterine evacuation. Single agent antimicrobials do not provide adequate coverage for the array of organisms that may be involved and therefore are not indicated. A laparoscopy can be indicated if ectopic pregnancy is suspected, but it is unlikely in this case. Medical termination is not the best option since prompt evacuation of the uterus is indicated for septic abortion.

A 38-year-old G1P0 woman is admitted at 42 weeks gestation with an anencephalic infant for induction of labor. The attending physician decides not to monitor the baby's heart rate during labor because he would not intervene with a Cesarean section in the event of fetal distress or demise. The physician's action is justified by which one of the following concepts? A. Beneficence to the fetus B. Respect for autonomy to the patient C. Maleficence to the fetus D. Non-maleficence to the patient E. Justice for the patient

D. The non-malfeasance principle expresses the concept that professionals have a duty to protect the patient from harm. Since an anencephalic infant will not survive, performing a Cesarean section on this patient will cause her harm. Beneficence principle expresses the concept that professionals have a duty to act for the benefit of others, and, in this case, performing a Cesarean section will not benefit the fetus.

A 23-year-old G2P1 woman with six weeks amenorrhea presents with lower abdominal pain and vaginal bleeding. Her temperature is 102.0°F (38.9°C) and the cervix is 1 cm dilated. Uterus is eight-week size and tender. There are no adnexal masses. Urine pregnancy test is positive. What is the most likely diagnosis? A. Threatened abortion B. Missed abortion C. Normal pregnancy D. Septic abortion E. Ectopic Pregnancy

D. The patient has a septic abortion. She has fever and bleeding with a dilated cervix which are findings seen with septic abortion. Threatened abortions clinically have vaginal bleeding, a positive pregnancy test and a cervical os closed or uneffaced. Missed abortions have retention of a nonviable intrauterine pregnancy for an extended period of time (i.e. dead embryo or blighted ovum). A normal pregnancy would have a closed cervix. Ectopic pregnancy would likely present with bleeding, abdominal pain, possibly have an adnexal mass, and the cervix would typically be closed.

A 32-year-old G0 woman comes to your office because she has been unable to conceive for one year. She is currently in a mutually monogamous relationship with her husband, has intercourse three times per week, and has no dyspareunia. Her menstrual cycles occur every 26-34 days. She has had seven sexual partners in the past. She was treated for multiple sexually transmitted infections including gonorrhea, chlamydia and pelvic inflammatory disease in her early twenties. She had an abnormal Pap test about four years ago and was treated with a LEEP. What is the most likely underlying cause of infertility in this patient? A. Luteal phase defect B. Cervical stenosis C. Ovulatory dysfunction D. Tubal disease E. Endometriosis

D. The rate of tubal infertility has been reported as 12% after one episode of PID, 25% after 2 episodes and 50% after three episodes. Salpingitis can develop in 15-30% of women with inadequately treated gonococcal or chlamydial infections and can result in significant long-term sequelae, such as chronic pelvic pain, hydrosalpinx, tubal scarring and ectopic pregnancy. Given this patient's history, her inability to conceive is most likely due to the long-term sequelae of a sexually transmitted infection. Although the patient had a LEEP, risk for cervical stenosis is low. She is having regular cycles; therefore, anovulation and luteal phase defect is less likely. This case emphasizes the importance of aggressive screening and treatment protocols for sexually transmitted infections, as well as counseling regarding abstinence and safer sex practices. While endometriosis can cause tubal occlusion, her clinical presentation is not consistent with endometriosis.

A 26-year-old G0 woman presents with severe right lower quadrant pain associated with nausea for the last six hours, which began shortly after she finished her aerobic exercises. She has a history of suspected endometriosis, which was diagnosed two years ago, based on her severe dysmenorrhea. She has been using NSAIDs during her menses to control the pain. She is not sexually active, and is otherwise in good health. Her menstrual cycles are regular and her last menstrual period was three weeks ago. She has no history of sexually transmitted infections. Her vital signs are: blood pressure 145/70; pulse 100; temperature 99.2°F (37.3°C). She appears uncomfortable. On abdominal examination, she has moderate tenderness to palpation in the right lower quadrant. On pelvic exam, she has no lesions or discharge. A thorough bimanual examination is difficult to perform due to her discomfort. Beta-hCG <5 mIU/ml and WBC 8,500 /microliter. A pelvic ultrasound shows a 6 cm right ovarian mass. The uterus and left ovary appear normal. There is a moderate amount of free fluid in the pelvis. What is the most likely diagnosis in this patient? A. Appendicitis B. Exacerbation of the endometriosis C. Ovarian carcinoma D. Ovarian torsion E. Ectopic pregnancy

D. The sudden onset of pain and nausea, as well as the presence of a cyst on ultrasound, suggest ovarian torsion. Although appendicitis is in the differential diagnosis, it is unlikely to have such a sudden onset of pain and a normal white count. Her endometriosis can get worse but it would be unlikely to be of such sudden onset. Although she has an adnexal mass, a negative Beta-hCG rules out pregnancy.

A 23-year-old G1P0 woman at 10 weeks gestation presents with an intrauterine embryonic demise. On exam, her blood pressure is 120/80; heart rate is 67; and she is afebrile. Her cervix is closed and there is no evidence of bleeding. She desires to have the most minimally invasive treatment as possible. Which of the following options is best for this patient? A. Dilation and curettage B. Manual vacuum aspiration C. Mifepristone D. Misoprostol E. Oxytocin

D. This patient has a missed abortion. Expectant management is the least invasive treatment. Of the options listed, Misoprostol (prostaglandin E1) is the least invasive best option for this patient. Misoprostol can be administered orally or vaginally and will induce uterine cramping with expulsion of products of conception. Potential risk factors of use include hemorrhage as well as failure. Dilation and curettage and manual vacuum aspiration are effective methods for treatment of a missed abortion, but are invasive procedures. Mifepristone, a progesterone receptor blocker, is used for pregnancy termination. It is recommended for use within 49 days of the last menstrual period, but there is data to show that it can be effective up to nine weeks. Oxytocin would not be effective as there has not been up-regulation of receptors at this gestational age.

A 56-year-old G3P3 woman presents to the office for her annual health maintenance examination. She is in good health and is not taking any medications. She has been postmenopausal for three years. She had an abnormal Pap test 10 years ago, but results have been normal every year since. She is sexually active with her husband. On examination, her cervix is 1 cm above the vaginal introitus and there is moderate bladder prolapse. Her uterus is normal in size and she has no adnexal masses or tenderness. In addition to recommending a mammogram, what is the most appropriate next step in the management of this patient? A. Cystocele repair B. Pelvic ultrasound C. Total hysterectomy D. Observation E. Topical estrogen

D. This patient is asymptomatic from her prolapse; therefore, no intervention is necessary at this point. Cystocele repairs and hysterectomies are invasive procedures which are not indicated in this asymptomatic patient. It is not necessary to obtain a pelvic ultrasound, as her uterus is normal in size and she has no adnexal masses. Topical estrogen would not help improve the prolapse, although it might help with her vaginal dryness. She seems to be doing well with the lubricants and it is not necessary to expose her to estrogen.

A 26-year-old G2P2 woman presents with urinary urgency and dysuria for the past three days. She has a history of a urinary tract infection once. She is sexually active and uses condoms for contraception. She is otherwise healthy and does not take any medications or supplements. She does not have fever, chills, flank pain or vaginal discharge. Which of the following organisms is the most likely cause of this patient's symptoms? A. Enterococcus faecalis B. Klebsiella pneumoniae C. Proteus mirabilis D. Staphylococcus saprophyticus E. Escherichia coli

E. Acute cystitis in a healthy, non-pregnant woman is considered uncomplicated and is very common. Escherichia coli causes 80 to 85 percent of cases. The other major pathogens are Staphylococcus saprophyticus, Klebsiella pneumoniae, Enterococcus faecalis and Proteus mirabilis. The physician must consider antibiotic resistance when determining treatment.

A 29-year-old G3P0 woman presents for evaluation and treatment of pregnancy loss. Her past medical history is remarkable for three early (<14 weeks gestation) pregnancy losses. Parental karyotype was normal. Which of the following is the most appropriate next step in the management of this patient? A. Prophylactic cerclage with her next pregnancy B. Serial cervical length with her next pregnancy C. 17-hydroxyprogesterone with her next pregnancy D. Check for Factor V Leiden mutation E. Check antiphospholipid antibodies

E. Antiphosphospholipid antibodies are associated with recurrent pregnancy loss. The workup for antiphospholipid syndrome includes assessment of anticardiolipin and beta-2 glycoprotein antibody status, PTT, and Russell viper venom time. Recurrent pregnancy loss is defined as > two consecutive or > three spontaneous losses before 20 weeks gestation. Etiologies include anatomic causes, endocrine abnormalities such as hyper- or hypothyroidism and luteal phase deficiency, parental chromosomal anomalies, immune factors such as lupus anticoagulant and idiopathic factors. Her history is not consistent with cervical insufficiency which is diagnosed in the second trimester by history, physical exam and other diagnostic tests, such as ultrasound. Serial cervical lengths or placement of a cerclage are not indicated in this patient. Treatment with 17-hydroxyprogesterone is indicated in patients with a history of prior preterm birth. Factor V Leiden mutation has not been associated with recurrent pregnancy loss. It can be associated with thrombotic events.

A 32-year-old G3P3 woman comes to the office to discuss permanent sterilization. She has a history of hypertension and asthma (on corticosteroids). She has been married for 10 years. Vital signs show: blood pressure 140/90; weight 280 pounds; height 5 feet 9 inches; and BMI 41.4kg/m2. You discuss with her risks and benefits of contraception. Which of the following would be the best form of permanent sterilization to recommend for this patient? A. Laparoscopic bilateral tubal ligation B. Mini laparotomy tubal ligation C. Exploratory laparotomy with bilateral salpingectomy D. Total abdominal hysterectomy E. Vasectomy for her husband

E. Both vasectomy and tubal ligation are 99.8% effective. Vasectomies are performed as an outpatient procedure under local anesthesia, while tubal ligations are typically performed in the operating room under regional or general anesthesia; therefore carrying slightly more risk to the woman, assuming both are healthy. She is morbidly obese, so the risk of anesthesia and surgery are increased. In addition, she has chronic medical problems that put her at increased risk of having complications from surgery.

A 28-year-old G2P2 woman returns today for follow up on her abnormal Pap test which reveals atypical squamous cells of undetermined significance (ASCUS). Reflex HPV testing is positive for high risk type. She has never had a prior abnormal Pap test, and has been following the recommended screening guidelines. She is asymptomatic. Her pelvic exam reveals a normal cervix with a small amount of cervical mucous. What is the next best step in the management of this patient? A. Routine screening B. Repeat Pap test in one year C. Repeat HPV testing in one year D. Repeat co-testing with Pap and HPV in one year E. Colposcopy

E. Colposcopy is indicated for all abnormal Pap test results including ASCUS Pap test when HPV is positive. Reflex HPV testing for high-risk DNA types should be performed in patients with ASCUS. If negative, then co-testing with cytology and HPV can be repeated in three years. Repeat cytology in one year is also an acceptable option for ASCUS if HPV testing cannot be done. http://www.asccp.org/Portals/9/docs/ASCCP%20Updated%20Guidelines%20Algorithms%206.3.13.pdf

A 26-year-old G0 women returns for a follow-up visit regarding suspected endometriosis. She has been using NSAIDs and oral contraceptive pills to help manage her pelvic pain which has been getting worse. While discussing further treatment options, she asks if there is any test or procedure you can perform to confirm her diagnosis. Which of the following would you recommend? A. CA-125 B. Pelvic ultrasound C. CT scan of the abdomen and pelvis D. MRI of the pelvis E. Diagnostic laparoscopy

E. Definitive diagnosis is based on exploratory surgery and biopsies, although endometriosis is usually initially treated based on the clinical presentation. In addition, this patient can benefit from laparoscopy, since she has failed the two most common treatments for endometriosis, NSAIDs and OCPs. There is no imaging study or blood test that can confirm the diagnosis of endometriosis. CA-125 is non-specific and can be elevated in patients with endometriosis, and therefore not helpful.

A 23-year-old G0 woman with last menstrual period 14 days ago presents to the office because she had unprotected intercourse the night before. She does not desire pregnancy at this time and is requesting contraception. She has no medical problems and is not taking any medications. In addition to offering her counseling and testing for sexually transmitted infections, which of the following is the most appropriate next step in the management of this patient? A. Observation for two weeks to establish if pregnancy occurred before initiating treatment B. Oral contraceptives now C. Oral contraceptives after her next normal menstrual period D. Emergency contraception and follow-up after next menstrual period E. Provide emergency contraception, then begin oral contraceptives immediately

E. Emergency contraceptive pills are not an abortifacient, and they have not been shown to cause any teratogenic effect if inadvertently administered during pregnancy. They are more effective the sooner they are taken after unprotected intercourse, and it is recommended that they be started within 72 hours, and no later than 120 hours. Plan B®, the levonorgestrel pills, can be taken in one or two doses and cause few side effects. Emergency contraceptive pills may be used anytime during a woman's cycle, but may impact the next cycle, which can be earlier or later with bleeding ranging from light, to normal, to heavy.

A 35-year-old G3P3 woman requests contraception. Her youngest child is seven years old. Her periods have been regular since she discontinued breastfeeding five years ago. Her past medical history includes depression that is controlled with antidepressants, and a history of deep venous thrombosis. She denies smoking or alcohol use. In the past, oral contraceptive pills have caused her to have severe gastrointestinal upset. What in her history makes her an ideal candidate for progestin-only pills? A. Depression B. Smoking history C. Severe nausea on combined oral contraceptives D. Lactation history E. Deep venous thrombosis

E. Ideal candidates for progestin-only pills include women who have contraindications to using combined oral contraceptives (estrogen and progestin containing). Contraindications to estrogen include a history of thromboembolic disease, women who are lactating, women over age 35 who smoke or women who develop severe nausea with combined oral contraceptive pills. Progestins should be used with caution in women with a history of depression.

A 24-year-old G1P1 woman comes to the office requesting contraception. Her past medical history is unremarkable, except for a family history of ovarian cancer. She denies alcohol, smoking and recreational drug use. She is in a monogamous relationship. She wants to decrease her risk of gynecological cancer. Of the following, what is the best method of contraception for this patient? A. Diaphragms B. Condoms C. Copper containing intrauterine device D. Progesterone containing intrauterine device E. Combined oral contraceptives

E. Oral contraceptives will decrease a woman's risk of developing ovarian and endometrial cancer. The first developed higher dose oral contraceptive pills have been linked to a slight increase in breast cancer, but not the most recent (current) lower dose pills. Women who use oral contraceptive pills have a slightly higher risk of developing cervical intraepithelial neoplasia, but their risk of developing PID, endometriosis, benign breast changes and ectopic pregnancy are reduced. Both hypertension and thromboembolic disorders can be a potential side effect from using oral contraceptive pills. Diaphragms, condoms and intrauterine devices will not lower her risk of ovarian cancer. The progesterone IUD may decrease a woman's risk for endometrial cancer but would not effect her risk for ovarian cancer, and have been associated with increased ovarian cysts.

A 35-year-old G3P3 woman comes to the office because she desires contraception. Her past medical history is significant for Wilson's disease, chronic hypertension and anemia secondary to menorrhagia. She is currently on no medications. Her vital signs reveal a blood pressure of 144/96. Which of the following contraceptives is the best option for this patient? A. Progestin-only pill B. Low dose combination contraceptive C. Continuous oral contraceptive D. Copper containing intrauterine device E. Levonorgestrel intrauterine device

E. The levonorgestrel intrauterine device has lower failure rates within the first year of use than does the copper containing intrauterine device. It causes more disruption in menstrual bleeding, especially during the first few months of use, although the overall volume of bleeding is decreased long-term and many women become amenorrheic. The levonorgestrel intrauterine device is protective against endometrial cancer due to release of progestin in the endometrial cavity. She is not a candidate for oral contraceptive pills because of her poorly controlled chronic hypertension. The progestin-only pills have a much higher failure rate than the progesterone intrauterine device. She is not a candidate for the copper-containing intrauterine device because of her history of Wilson's disease.

A 17-year-old G0 sexually active female presents to the emergency department with pelvic pain that began 24 hours ago. She reports menarche at the age of 15 and coitarche soon thereafter. She has had four male partners, including her new boyfriend of a few weeks. Her blood pressure is 100/60; pulse 100; and temperature 102.0°F (38.9°C). On speculum examination, you note a foul-smelling mucopurulent discharge from her cervical os and she has significant tenderness with manipulation of her uterus. What is the next best step in the management of this patient? A. Outpatient treatment with oral broad spectrum antibiotics B. Outpatient treatment with intramuscular and oral broad spectrum antibiotics C. Intravenous antibiotics and dilation and curettage D. Inpatient treatment, laparoscopy with pelvic lavage E. Inpatient treatment and intravenous antibiotics

E. The most likely cause of the symptoms and signs in this patient is infection with a sexually transmitted organism. The most likely organisms are both N. gonorrhoeae and chlamydia, and the patient should be treated empirically for both after appropriate blood and cervical cultures are obtained. There is no evidence that adolescents have better outcomes from inpatient therapy. However, since the patient also has a high fever, inpatient admission is recommended for aggressive intravenous antibiotic therapy in an effort to prevent scarring of her fallopian tubes and possible future infertility.

A 16-year-old G0 female presents to the emergency department with a two-day history of abdominal pain. She is sexually active with a new partner and is not using any form of contraception. Temperature is 101.8°F (38.8°C). On examination, she has lower abdominal tenderness and guarding. On pelvic exam, she has diffuse tenderness over the uterus and bilateral adnexal tenderness. Beta-hCG is <5. What is the most likely diagnosis for this patient? A. Ectopic pregnancy B. Appendicitis C. Acute cystitis D. Endometriosis E. Acute salpingitis

E. The signs and symptoms of acute salpingitis can vary and be very subtle with mild pain and tenderness, or the patient can present in much more dramatic fashion with high fever, mucopurulent cervical discharge and severe pain. Important diagnostic criteria include lower abdominal tenderness, uterine/adnexal tenderness and mucopurulent cervicitis.

A 24-year-old G0 woman presents with multiple painful ulcers involving the vulva. The sores were initially fluid filled, but are now open, weeping and crusted. She reports a fever and is having difficulty voiding due to pain. She uses a vaginal ring for contraception. She has multiple sexual partners and uses condoms for vaginal intercourse. She is distraught that she may have a sexually transmitted infection. She is healthy and does not smoke or use drugs. On physical exam, she is in obvious distress. Temperature is 100.2°F (37.9°C); pulse 100. Examination of the genital tract is limited due to her discomfort. Multiple ulcers and erosions of variable size are localized to the perineum, labia minora and vestibule. Swelling is diffuse. The lesions are eroded, some with a purulent eschar. There is exquisite tenderness to touch. What further testing should be offered to this patient? A. RPR (rapid plasma regain) B. HIV C. Herpes culture D. DNA probe for gonorrhea and chlamydia E. All of the above

E. This patient has classic primary herpes with painful genital ulcerations, fever and dysuria. Given the presence of one sexually transmitted infection, screening should be offered for other STIs. Resolution of the acute episode is required before a speculum can be inserted to allow endocervical sampling for gonorrhea and chlamydia. If it was a high-risk exposure, prophylactic empiric treatment could be offered to cover gonorrhea and chlamydia. The patient should be counseled that primary herpes can be acquired despite condoms and even by oral-genital inoculation. Hepatitis B vaccination should be offered to protect her against any future exposures. She should be encouraged to discuss her diagnosis with all sexual partners and to continue to reliably use latex condoms.

A 23-year-old G1P0 woman at six weeks gestation undergoes a medical termination of pregnancy. One day later, she presents to the emergency department with bleeding and soaking more than a pad per hour for the last five hours. Her blood pressure on arrival is 110/60; heart rate 86. On exam, her cervix is 1 cm dilated with active bleeding. Hematocrit on arrival is 29%. Which of the following is the most appropriate next step in the management of this patient? A. Admit for observation B. Repeat hematocrit in six hours C. Begin transfusion with O-negative blood D. Give an additional dose of prostaglandins E. Perform a dilation and curettage

E. This patient is having heavy bleeding as a complication of medical termination of pregnancy. The most likely etiology for her bleeding is retained products of conception. This is managed best by performing a dilation and curettage. It is not appropriate to wait six hours before making a decision regarding next step in management, or to just admit her for observation. Since the patient is not symptomatic from her anemia, it is not necessary to transfuse her at this time.

A 26-year-old G0 presents to the emergency department with eight hours of severe right lower quadrant pain associated with nausea. She has a history of suspected endometriosis, which was diagnosed two years ago based on severe dysmenorrhea. She has been using NSAIDs during her menses to control the pain. She is not sexually active. She is otherwise in good health. Her menstrual cycles are regular and her last menstrual period was three weeks ago. She has no history of sexually transmitted infections. Her vital signs are: blood pressure 145/70; pulse 100; temperature 98.6°F (37.0°C). She appears uncomfortable. On abdominal examination, she has moderate tenderness to palpation in the right lower quadrant. On pelvic examination, she has no lesions or discharge. A thorough bimanual examination was difficult to perform due to her discomfort. Beta-hCG <5 mIU/ml and hematocrit 29%. A pelvic ultrasound shows a 6 cm right ovarian mass. The uterus and left ovary appear normal. There is a moderate amount of free fluid in the pelvis. What is the most appropriate next step in the management of this patient? A. Begin oral contraceptives B. MRI of the pelvis C. Doppler pelvic ultrasound D. CT scan of the pelvis E. Surgical exploration

E. This patient most likely has ovarian torsion and needs to be surgically explored. Further imaging studies will not help beyond the information obtained on the ultrasound. A Doppler ultrasound to check the blood flow to the ovaries is controversial, as normal flow does not rule out ovarian torsion. Although oral contraceptives can help decrease the development of further cyst formation and control the pain associated with endometriosis, this patient needs immediate surgical attention due to suspected ovarian torsion.


Conjuntos de estudio relacionados

Final exam-Virginia life and Health

View Set

International Business Section G1 Test #1

View Set

Network+ Chapter 5 IPv4 and IPv6 Addresses

View Set

Social Psychology Final Exam Study Guide 9 and 10

View Set

Macro-Econ:Ch1 Limits, Alternatives and Choices

View Set

Emberi Erőforrások A. tételsor/ 1. tétel

View Set

Colonial Latin American Social Classes

View Set

Ch. 14/15/16/30 Peri-op and Post-op care

View Set